Priapism: Difference between revisions

From UrologySchool.com
Jump to navigation Jump to search
 
(95 intermediate revisions by the same user not shown)
Line 1: Line 1:
'''To be updated with [https://pubmed.ncbi.nlm.nih.gov/35536142/ 2022 AUA/SMSNA Guidelines on Priapism]'''
'''Includes [https://pubmed.ncbi.nlm.nih.gov/35536142/ 2021 AUA/SMSNA Guidelines on Priapism]'''
 
* Guideline search up to February 2021
See [https://www.youtube.com/watch?v=AEr09ZYQPQE Video Review on AUA Guidelines on Priapism]


== Definition ==
== Definition ==


* '''<span style="color:#ff0000">Priapism: a persistent penile erection that continues > 4 hours beyond, or is unrelated to, sexual stimulation[https://pubmed.ncbi.nlm.nih.gov/35536142/ ★]</span>'''
* '''<span style="color:#ff0000">Priapism: a persistent penile erection that continues > 4 hours beyond, or is unrelated to, sexual stimulation[https://pubmed.ncbi.nlm.nih.gov/35536142/ ★]</span>'''
*Typically, only the corpora cavernosa are affected.


== Classification (3) ==
== Classification (3) ==


# '''<span style="color:#ff0000">Ischemic</span>'''
# '''<span style="color:#ff0000">Ischemic</span>'''
# '''<span style="color:#ff0000">Stuttering</span>'''
# '''<span style="color:#ff0000">Non-ischemic</span>'''
# '''<span style="color:#ff0000">Non-ischemic</span>'''
#'''<span style="color:#ff0000">Recurrent non-ischemic (stuttering)</span>'''


=== Ischemic priapism (veno-occlusive, low-flow) ===
=== Ischemic priapism (veno-occlusive, low-flow) ===
* '''<span style="color:#ff0000">Majority of cases</span>'''
* '''<span style="color:#ff0000">Majority of cases</span>'''
* '''<span style="color:#ff0000">Associated with</span>''' (features that distinguish this from non-ischemic priapism)
* '''Characterized by little or no cavernous blood flow'''
*'''<span style="color:#ff0000">Associated with</span>''' (features that distinguish this from non-ischemic priapism)
*# '''<span style="color:#ff0000">Pain</span>'''
*# '''<span style="color:#ff0000">Pain</span>'''
*# '''<span style="color:#ff0000">Rigid corpora cavernosa</span>'''
*# '''<span style="color:#ff0000">Fully rigid and tender corpora cavernosa</span>'''
*# '''<span style="color:#ff0000">Abnormal blood gas</span>'''
*# '''<span style="color:#ff0000">Abnormal blood gas</span> (i.e., hypoxic, hypercarbic, acidotic)'''
* '''<span style="color:#ff0000">Once properly diagnosed, requires emergent intervention</span>'''
*#'''<span style="color:#ff0000">Requires prompt evaluation and may require emergency management</span>'''
** '''<span style="color:#ff0000">Duration of ischemic priapism is associated with the risk of future erectile dysfunction</span>'''
*#'''<span style="color:#ff0000">Non-traumatic etiology</span>'''
*** In sickle cell disease patients in whom priapism was reversed, spontaneous erections (with or without use of sildenafil) were reported in:
* Natural history of untreated acute ischemic priapism includes days to weeks of painful erections followed by permanent loss of erectile function
**** '''100% when priapism was reversed by 12 hours'''
*'''<span style="color:#ff0000">Duration of ischemic priapism is associated with the risk of future erectile dysfunction</span>'''
**** '''78% when reversed by 12-24 hours'''
** In sickle cell disease patients in whom priapism was reversed, spontaneous erections (with or without use of sildenafil) were reported in:
**** '''44% when reversed by 24-36 hours'''
*** '''100% when priapism was reversed by 12 hours'''
**** '''0% when reversed ≥36 hours'''
*** '''75% when reversed by 12-24 hours'''
*** '''A more recent study that the cutoff for irreversible restoration of erectile tissue is 48 hours'''
*** '''≈50% when reversed by 24-36 hours'''
*** '''<span style="color:#ff0000">Interventions beyond 48-72 hours of onset may relieve erection and pain but have little benefit in preserving potency</span>'''
*** '''0% when reversed ≥36 hours'''
 
***[https://pubmed.ncbi.nlm.nih.gov/18312286/ Bennett, Nelson, and John Mulhall. "Sickle cell disease status and outcomes of African-American men presenting with priapism." ''The journal of sexual medicine'' 5.5 (2008): 1244-1250.]
=== Stuttering priapism (intermittent) ===
** '''A more recent study suggested that the cutoff for irreversible restoration of erectile tissue is 48 hours[https://pubmed.ncbi.nlm.nih.gov/23892191/]'''
* '''<span style="color:#ff0000">Transient prolonged erections lasting < 2 hours</span>'''
** '''<span style="color:#ff0000">Interventions beyond 48-72 hours of onset may relieve erection and pain but have little benefit in preserving potency</span>'''
* '''<span style="color:#ff0000">Characterized by a pattern of recurrence</span>'''
* '''<span style="color:#ff0000">Commonly associated with sickle cell disease</span>'''
** '''<span style="color:#ff0000">Priapism in children and adolescents is most commonly related to sickle cell disease</span>'''
*** The sickle cell genetic mutation is the result of a single amino acid substitution in the beta-globin subunit of hemoglobin S.
*** Clinical features seen in homozygous sickle cell disease: chronic hemolysis, vascular occlusion, tissue ischemia, and end-organ damage.
*** Sickle cell trait can be a predisposing factor to ischemic priapism.
** Pathogenesis is related to hemolysis and reduced nitric oxide
* '''<span style="color:#ff0000">Any patient who has experienced an episode of ischemic priapism is also at risk for stuttering priapism</span>'''


=== Non-ischemic priapism (arterial, high-flow) ===
=== Non-ischemic priapism (arterial, high-flow) ===
* '''<span style="color:#ff0000">Relatively rare</span>'''
* '''<span style="color:#ff0000">Relatively rare</span>'''
* '''<span style="color:#ff0000">Pathogenesis:</span>'''
* May last hours to weeks and is frequently recurrent
** '''<span style="color:#ff0000">Usually involves perineal or penile trauma</span>''' resulting in laceration of the cavernous artery (or one of its branches within the corpora) '''and unregulated cavernous arterial inflow. The arteriolacunar fistula is typically unilateral'''.
*'''<span style="color:#ff0000">Pathogenesis:</span>'''
** Likely results from unregulated control of arterial inflow and cavernous smooth muscle tone
**'''<span style="color:#ff0000">Usually involves perineal or penile trauma</span>''' '''resulting in laceration of the cavernous artery (or one of its branches within the corpora)''' '''and unregulated cavernous arterial inflow.'''
*** '''<span style="color:#ff0000">Most common cause is a straddle injury</span>''' to the crura
*** '''<span style="color:#ff0000">Most common cause is a straddle injury</span>''' to the crura
**** '''Other mechanisms of traumatic arterial laceration include''' coital trauma, kicks to the penis or perineum, pelvic fractures, birth canal trauma to the male newborn, needle lacerations, iatrogenic needle injury, complications of penile diagnostics, vascular erosions complicating metastatic infiltration of the corpora, and after iatrogenic trauma from coldknife urethrotomy, corporoplasty, and penile revascularization procedures
**** '''Other mechanisms of traumatic arterial laceration include''' coital trauma, kicks to the penis or perineum, pelvic fractures, birth canal trauma to the male newborn, needle lacerations, iatrogenic needle injury, complications of penile diagnostics, vascular erosions complicating metastatic infiltration of the corpora, and after iatrogenic trauma from coldknife urethrotomy, corporoplasty, and penile revascularization procedures
***** '''Sustained partial erection may develop 24 hours after perineal or penile blunt trauma'''
***** '''Sustained partial erection may develop 24 hours after perineal or penile blunt trauma'''
***** '''<span style="color:#ff0000">After either aggressive medical management of ischemic priapism or surgical shunting, priapism may rapidly recur with conversion from ischemic (low-flow) to non-ischemic (high-flow)'''
***'''The arteriolacunar fistula is typically unilateral'''.
** '''<span style="color:#ff0000">After either aggressive medical management of ischemic priapism or surgical shunting, priapism may rapidly recur with conversion from ischemic (low-flow) to non-ischemic (high-flow)'''
** '''Other causes include Fabry disease and sickle cell anemia'''
** '''Other causes include Fabry disease and sickle cell anemia'''
* '''<span style="color:#ff0000">Associated with</span>''' (features that distinguish this from ischemic priapism)
* '''<span style="color:#ff0000">Associated with</span>''' (features that distinguish this from ischemic priapism)
Line 54: Line 52:
*#* '''Patients do report additional engorgement with sexual stimulation, with return to partial erection after climax'''
*#* '''Patients do report additional engorgement with sexual stimulation, with return to partial erection after climax'''
*# '''<span style="color:#ff0000">Normal penile blood gas</span>''' (features that distinguish this from non-ischemic priapism)
*# '''<span style="color:#ff0000">Normal penile blood gas</span>''' (features that distinguish this from non-ischemic priapism)
* '''<span style="color:#ff0000">Once properly diagnosed, does not require emergent intervention</span>'''
*#'''<span style="color:#ff0000">Once properly diagnosed, does not require emergent intervention</span>'''
*#'''<span style="color:#ff0000">Traumatic etiology, usually</span>'''
 
=== Recurrent ischemic priapism ===
* Also known as "stuttering" priapism
*'''<span style="color:#ff0000">Definition: Recurrent ischemic episodes, with or without meeting the 4-hour time criteria for priapism</span>'''
**Unwanted painful erections occur repeatedly with intervening periods of detumescence
**The key differentiating factor between the current definition of recurrent ischemic priapism and other recurrent priapism-like conditions is the requirement of confirmed penile ischemia.
***Priapism-like conditions include sleep-related painful erections, undesired prolonged erections, and recurrent non-ischemic priapism
*'''Both acute ischemic priapism and non-ischemic priapism may recur over time'''
*'''<span style="color:#ff0000">Commonly associated with sickle cell disease</span>'''
** '''<span style="color:#ff0000">Priapism in children and adolescents is most commonly related to sickle cell disease</span>'''
*** The sickle cell genetic mutation is the result of a single amino acid substitution in the beta-globin subunit of hemoglobin S.
*** Clinical features seen in homozygous sickle cell disease: chronic hemolysis, vascular occlusion, tissue ischemia, and end-organ damage.
*** '''Sickle cell trait can be a predisposing factor to ischemic priapism.'''
** Pathogenesis is related to hemolysis and reduced nitric oxide
* '''<span style="color:#ff0000">Management focuses on</span>'''
**'''<span style="color:#ff0000">Treatment of acute episodes</span>'''
**'''<span style="color:#ff0000">Prevention of future events</span>'''


== Causes (8) ==
*


# '''<span style="color:#ff0000">Medications (9):</span>'''
== Pathogenesis ==
## '''<span style="color:#ff0000">α-blockers (prazosin, terazosin, doxazosin, tamsulosin)</span>'''
 
=== Risk factors (8) ===
#'''<span style="color:#ff0000">Medications (9):</span>'''
## '''<span style="color:#ff0000">α-blockers (tamsulosin, terazosin, doxazosin, prazosin)</span>'''
## '''<span style="color:#ff0000">Anti-anxiety agents</span> (hydroxyzine)'''
## '''<span style="color:#ff0000">Anti-anxiety agents</span> (hydroxyzine)'''
## '''<span style="color:#ff0000">Anti-depressants and antipsychotics</span> (trazodone, bupropion, fluoxetine, sertraline, lithium, clozapine, risperidone, olanzapine, chlorpromazine, thioridazine, phenothiazines)'''
## '''<span style="color:#ff0000">Anti-depressants and antipsychotics</span> (trazodone, bupropion, fluoxetine, sertraline, lithium, clozapine, risperidone, olanzapine, chlorpromazine, thioridazine, phenothiazines)'''
## '''<span style="color:#ff0000">Anti-coagulants</span> (heparin, warfarin)'''
## '''<span style="color:#ff0000">Anti-coagulants</span> (heparin, warfarin)'''
## '''<span style="color:#ff0000">Antihypertensives </span>(hydralazine, guanethidine, propranolol)'''
## '''<span style="color:#ff0000">Anti-hypertensives </span>(hydralazine, propranolol,''' guanethidine''')'''
## '''<span style="color:#ff0000">Attention-deficit/hyperactivity disorder agents (methylphenidates</span>''' (concerta, daytrana, focalin, metadate, methylin, quillivant, ritalin)''', atomoxetine (strattera))'''
## '''<span style="color:#ff0000">Attention-deficit/hyperactivity disorder agents (methylphenidates</span>''' (concerta, daytrana, focalin, metadate, methylin, quillivant, ritalin)''', atomoxetine (strattera))'''
## '''<span style="color:#ff0000">Recreational drugs (alcohol, cocaine</span>''' (intranasal and topical)''', crack cocaine, marijuana)'''
## '''<span style="color:#ff0000">Recreational drugs (alcohol, cocaine</span>''' (intranasal and topical)''', <span style="color:#ff0000">marijuana)</span>'''
## '''<span style="color:#ff0000">Hormones (gonadotropin-releasing hormone, testosterone)</span>'''
## '''<span style="color:#ff0000">Hormones (testosterone, gonadotropin-releasing hormone)</span>'''
## '''<span style="color:#ff0000">Vasoactive erectile agents (papaverine, phentolamine, prostaglandin E1, oral phosphodiesterase type 5 (PDE5) inhibitors, combination intracavernous therapy)</span>'''
## '''<span style="color:#ff0000">Vasoactive erectile agents (oral phosphodiesterase type 5 (PDE5) inhibitors, papaverine, phentolamine, prostaglandin E1, combination intracavernous therapy)</span>'''
##* Most case reports describing priapism after use of a PDE5 inhibitor reveal histories of increased risk of priapism: sickle cell disease, spinal cord injury, use of a PDE5 inhibitor recreationally, use of a PDE5 inhibitor in combination with intracavernosal injections, history of penile trauma, use of psychotropic medications, or use of recreational drugs; '''risk is even lower for daily dosing PDE5 inhibitor'''
##* Most case reports describing priapism after use of a PDE5 inhibitor reveal histories of increased risk of priapism: sickle cell disease, spinal cord injury, use of a PDE5 inhibitor recreationally, use of a PDE5 inhibitor in combination with intracavernosal injections, history of penile trauma, use of psychotropic medications, or use of recreational drugs; '''risk is even lower for daily dosing PDE5 inhibitor'''
# '''<span style="color:#ff0000">Genitourinary conditions </span>(straddle injury, coital injury, pelvic trauma, kick to penis or perineum, arteriovenous or arteriocavernous bypass surgery, urinary retention)'''
# '''<span style="color:#ff0000">Genitourinary trauma </span>(straddle injury, pelvic trauma, coital injury, kick to penis or perineum)'''
# '''<span style="color:#ff0000">Thrombotic disease states </span>(asplenia, erythropoietin use, hemodialysis with heparin use, and cessation of warfarin)'''
#'''<span style="color:#ff0000">Genitourinary conditions </span>(urinary retention''', arteriovenous or arteriocavernous bypass surgery''')'''
# '''<span style="color:#ff0000">Hematologic dyscrasias (sickle cell disease, thalassemia, granulocytic leukemia, myeloid leukemia, lymphocytic leukemia, multiple myeloma, hemoglobin Olmsted variant, fat emboli associated with hyperalimentation, hemodialysis, glucose-6-phosphate dehydrogenase deficiency)</span>'''
# '''<span style="color:#ff0000">Thrombotic diseases/states </span>(thrombophilc states''' (deficiencies of protein C, S or Factor V Leiden), asplenia, erythropoietin use, hemodialysis with heparin use, and cessation of warfarin''')'''
# '''<span style="color:#ff0000">Hematologic diseases (sickle cell disease, thalassemia, hemolytic anemias (congenital dyserythropoietic anemia type II, unstable hemoglobinopathies, polycythemia, thrombotic thrombocytopenic purpura, multiple myeloma, chronic myelogenous or lymphocytic leukemias)</span>'''
##In homozygous sickle cell anemia, the most common form of SCD, priapism occurs in 23-89% of males by age 18.[https://pubmed.ncbi.nlm.nih.gov/10598664/]
# '''<span style="color:#ff0000">Infectious (toxin-mediated) causes (scorpion sting, spider bite, rabies, malaria)</span>'''
# '''<span style="color:#ff0000">Infectious (toxin-mediated) causes (scorpion sting, spider bite, rabies, malaria)</span>'''
# '''<span style="color:#ff0000">Metabolic conditions </span>(amyloidosis, Fabry disease, gout)'''
# '''<span style="color:#ff0000">Metabolic conditions </span>(amyloidosis, Fabry disease, gout)'''
# '''<span style="color:#ff0000">Cancer </span>(metastatic or regional infiltration) (prostate, urethra, testis, bladder, rectum, lung, kidney)'''
# '''<span style="color:#ff0000">Cancer </span>(metastatic or regional infiltration) (prostate, urethra, testis, bladder, rectum, lung, kidney)'''
# '''<span style="color:#ff0000">Neurogenic conditions </span>(syphilis, spinal cord injury, autonomic neuropathy, lumbar disk herniation, spinal stenosis, cerebral vascular accident, brain tumor, spinal anesthesia, cauda equina syndrome)'''
# '''<span style="color:#ff0000">Neurogenic conditions </span>(spinal cord injury, cerebral vascular accident, brain tumor,''' syphilis, autonomic neuropathy, lumbar disk herniation, spinal stenosis, spinal anesthesia, cauda equina syndrome''')'''


== Molecular basis of ischemic and stuttering priapism ==
== Diagnosis and Evaluation ==


* '''Direct result of NO imbalance''' resulting in aberrant molecular signaling, '''PDE5 activity downregulation,''' adenosine overproduction, and reductions in Rho-kinase activity, '''translating into enhanced corporal smooth muscle relaxation and inhibition of vasoconstriction in the penis'''
* See [https://www.auanet.org/documents/Guidelines/PDF/priapism/NIP%20JU%20SUMMARY%20Figure%20One%20Diagnosis%20of%20Priapism.pdf AUA/SMSNA Guideline Flowchart on Diagnosis of Priapism]
* Excessive adenosine accumulation in the penis, coupled with increased A(2B)R signaling, contributes to priapism in two independent lines of mutant mice.
 
== Diagnosis and Evaluation ==


=== UrologySchool.com Summary ===
=== UrologySchool.com Summary ===
* '''<span style="color:#ff0000">Recommended</span>'''
* '''<span style="color:#ff0000">Recommended</span>'''
**'''<span style="color:#ff0000">History and Physical Exam</span>'''
**'''<span style="color:#ff0000">History and Physical Exam</span>'''
** '''<span style="color:#ff0000">Labs (1)</span>'''
** '''<span style="color:#ff0000">Labs (2)</span>'''
**# '''<span style="color:#ff0000">Corporal blood gas</span>'''
**# '''<span style="color:#ff0000">Corporal blood gas</span>'''
* '''<span style="color:#ff0000">Optional</span>'''
* '''<span style="color:#ff0000">Optional</span>'''
**'''<span style="color:#ff0000">Labs</span>'''
**'''<span style="color:#ff0000">Labs (3)</span>'''
***'''<span style="color:#ff0000">CBC</span>'''
**#'''<span style="color:#ff0000">CBC</span>'''
***'''<span style="color:#ff0000">Hemoglobin electrophoresis</span>'''
**#'''<span style="color:#ff0000">Hemoglobin electrophoresis</span>'''
***'''<span style="color:#ff0000">Screening for psychoactive drugs and urine toxicology</span>'''
**#'''<span style="color:#ff0000">Screening for psychoactive drugs and urine toxicology</span>'''
**'''<span style="color:#ff0000">Imaging</span>'''
**'''<span style="color:#ff0000">Imaging</span>'''
***'''<span style="color:#ff0000">Penile duplex Doppler ultrasonography</span>'''
***'''<span style="color:#ff0000">Penile duplex Doppler ultrasonography</span>'''
****When the diagnosis of acute ischemic versus non-ischemic priapism is indeterminate


=== History and Physical Exam ===
=== History and Physical Exam ===
* '''In the majority of cases, the differentiation of acute ischemic priapism versus non-ischemic priapism may be made using only the history and physical exam.''' 
** In cases where the subtype is indeterminate, additional testing may be warranted.


==== History ====
==== History ====
* '''Duration of erection, presence of pain'''
* '''<span style="color:#ff0000">Signs and Symptoms</span>'''
* '''PMHx: sickle cell disease, hemoglobinopathies, hypercoagulable states, trauma to the pelvis, perineum, or penis'''
**'''<span style="color:#ff0000">Sexual history</span>'''
* '''Medications'''
***'''Duration of erection, presence of pain'''
* '''Use of any erectogenic therapies (both prescription and nutritional supplements)'''
***'''Baseline erectile function'''
* '''Use of recreational drugs'''
* '''<span style="color:#ff0000">Risk factors</span>'''
* '''Previous episodes of priapism and method of treatment'''
**'''<span style="color:#ff0000">Medical history</span>'''
* '''Baseline erectile function'''
*** '''Personal or family history of sickle cell disease, hemoglobinopathies, hypercoagulable states'''
***'''Pelvic, genital, or perineal trauma, especially a perineal straddle injury'''
***'''Previous episodes of priapism and method of treatment'''
** '''<span style="color:#ff0000">Medications that might have precipitated the episode</span>'''
*** '''Use of any erectogenic therapies (both prescription and nutritional supplements)'''
*** '''Use of recreational drugs'''


==== Physical Exam ====
==== Physical Exam ====
* '''In ischemic priapism, the corporal bodies will be completely rigid; the glans penis and corpus spongiosum are not. The penis may be tender'''
* '''<span style="color:#ff0000">Genitalia</span>'''
* '''In non-ischemic priapism, the corporal bodies will be tumescent, partially erect, but not completely rigid. The penis is also unlikely to be tender'''
**'''<span style="color:#ff0000">The corpora cavernosa are typically affected while the corpus spongiosum and the glans penis are not</span>'''
**'''<span style="color:#ff0000">In ischemic priapism, the corpora cavernosa are often fully rigid and tender</span>'''
**'''<span style="color:#ff0000">In non-ischemic priapism, the corporal cavernosa will be tumescent, partially erect, but not completely rigid. The penis is also unlikely to be tender</span>'''
*'''<span style="color:#ff0000">Abdominal, pelvic, and perineum</span>'''
**'''May reveal evidence of trauma or malignancy.'''


=== Laboratory ===
=== Laboratory ===


* '''Corporal blood gas by aspiration'''
* '''<span style="color:#ff0000">Corporal blood gas by aspiration</span>'''
** '''Recommended in the emergency evaluation of priapism'''
** '''<span style="color:#ff0000">Should be obtained in the emergency evaluation of priapism</span>[https://pubmed.ncbi.nlm.nih.gov/35536142/ ★]'''
***Clinical situations where a blood gas may be omitted at the clinician’s discretion (3):
***#Priapism induced by in-office or at home ICI therapies
***#Cases of recurrent ischemic priapism (i.e., SCD)
***#When the diagnosis is abundantly clear by history and examination alone
**Most common diagnostic methods of distinguishing acute ischemic priapism from non-ischemic priapism when the diagnosis cannot be made by history alone
**Blood aspirated from the corpus cavernosum in patients with acute ischemic priapism is hypoxic (dark red), while corporal blood in NIP patients is normally oxygenated (bright red)


{| class="wikitable"
{| class="wikitable"
!Source
!Source
!PO2 (mm Hg)
!<span style="color:#ff0000">PO2 (mm Hg)</span>
!PCO2 (mm Hg)
!<span style="color:#ff0000">PCO2 (mm Hg)</span>
!pH
!<span style="color:#ff0000">pH</span>
|-
|-
!Normal aterial blood (room air)
!Normal arterial blood (room air)
|>90
|'''>90'''
|<40
|'''<40'''
|7.40
|'''7.40'''
|-
|-
!Normal mixed venous blood (room air)
!Normal mixed venous blood (room air)
Line 133: Line 170:
|7.35
|7.35
|-
|-
!Ischemic priapism (first corporal aspirate)
!<span style="color:#ff0000">Ischemic priapism (first corporal aspirate)</span>
|<30
|<span style="color:#ff0000">'''<30'''</span>
|>60
|<span style="color:#ff0000">'''>60'''</span>
|<7.25
|<span style="color:#ff0000">'''<7.25'''</span>
|}
|}


* '''CBC and coagulation profile (PT/INR)'''
* '''CBC'''
** Evaluates for anemia, infection, hematologic abnormalities, and ensures that the patient can safely tolerate surgical interventions, if needed
** May identify elevated white blood cell counts, potentially identifying cases where priapism is due to underlying malignancy (e.g., leukemia). 
* '''Sickle cell preparation and hemoglobin electrophoresis'''
**Among men with sickle cell disease, acute ischemic priapism is associated with lower hemoglobin and elevated lactate dehydrogenase, bilirubin, aspartate aminotransferase, reticulocyte count, white blood cells, and platelet counts.
** '''Should be requested in African-Americans'''
**Platelet and eosinophil counts may also be elevated in men with acute ischemic priapism.
* '''Hemoglobin electrophoresis and other sickle cell testing'''
** '''May be appropriate in select clinical scenarios and based on underlying clinical suspicion (e.g., patient race)'''
***In most cases, most men with sickle cell disease have been diagnosed previously. 
****The yield of identifying men with previously undiagnosed SCD among a cohort of men presenting with priapism is not well established. 
*'''Urine and serum toxicology panels'''
*'''Urine and serum toxicology panels'''
** '''Should be done if recreational narcotic or prescription psychoactive drugs are suspected from the history'''
** A thorough medication and social history may provide enough information to determine the underlying cause of the priapism without collection of these studies.
***Testing for potential substances may have a high rate of false negativity, particularly with synthetic and otherwise altered versions of common illicit substances
*'''While CBC, hemoglobin electrophoresis, and toxicology panels may contribute to the identification of underlying cause, they often will not be used to guide treatment of the acute presentation'''


=== Imaging ===
=== Imaging ===


* '''Color Doppler US'''
==== Color Doppler US ====
** '''Indications:'''
* '''<span style="color:#ff0000">Indications</span>'''
**# '''History suggests penile or perineal trauma or if the corporal aspirate reveals well-oxygenated blood'''
**'''<span style="color:#ff0000">Should not be incorporated into the acute evaluation and management of priapism in the emergency department by non-urologist specialists</span>[https://pubmed.ncbi.nlm.nih.gov/35536142/ ★]'''
**# '''Full or partial erection after treatments for ischemic priapism.'''
***Not the primary way to diagnose priapism
**#* The differential diagnosis includes resolved ischemia with penile edema, persistent ischemia, and conversion to high-flow state
**'''<span style="color:#ff0000">May be utilized in less clearly delineated cases to differentiate between acute ischemic priapism and non-ischemic priapism</span>[https://pubmed.ncbi.nlm.nih.gov/35536142/ ★]'''
** '''Should assess the ventral surface of the penis in a sagittal, para-urethral plane and perineum.'''
*Findings[https://pubmed.ncbi.nlm.nih.gov/34495686/ §]
* Penile arteriography
**Acute ischemic priapism
** Too invasive as a diagnostic procedure to differentiate ischemic from nonischemic priapism
***Bilateral absence of flow through the cavernosal arteries
* '''MRI'''
***Peak systolic flows <50 cm/sec
** '''Can be used to demonstrate:'''
***Mean velocity <6.5 cm/sec
**# '''A well-established arteriolar-sinusoidal fistula'''
***Diastolic reversal (i.e., negative end diastolic velocities)
**# '''Presence and extent of tissue thrombus'''
**Non-ischemic priapism
**# '''Corporal smooth muscle infarction'''
***Peak systolic velocities of >50 cm/sec.
**# '''Corporal malignancy or metastasis'''
***In the non-acute setting, may identify anatomical abnormalities, such as a cavernous artery fistula or pseudoaneurysm in patients who already have been diagnosed with non-ischemic priapism.
****These abnormalities may occur following a straddle injury or direct scrotal trauma and are, therefore, most often found in the perineal portions of the corpora cavernosa.
*Use is limited by
**Number of specialists who can currently perform the procedure
**Equipment might not be readily available
==== Pelvic MRI ====
* Likely does not have a role in the initial diagnostic and treatment phase of priapism'''<span style="color:#ff0000">[https://pubmed.ncbi.nlm.nih.gov/35536142/ ★]</span>'''
*'''Can be used to demonstrate:'''
*# '''A well-established arteriolar-sinusoidal fistula'''
*# '''Presence and extent of tissue thrombus'''
*# '''Corporal smooth muscle infarction'''
*# '''Corporal malignancy or metastasis'''
 
==== Penile arteriography ====
* Too invasive as a diagnostic procedure to differentiate ischemic from non-ischemic priapism


== Management ==
== Management ==


=== Ischemic priapism ===
=== Ischemic Priapism ===
See [https://www.auanet.org/documents/Guidelines/PDF/priapism/NIP%20JU%20SUMMARY%20Figure%20Two%20Treatment%20of%20Acute%20Ischemic%20Priapism.pdf AUA/SMSNA Guideline Flowchart on Management of Acute Ischemic Priapism]
 
==== Patient Counselling ====
 
* '''<span style="color:#ff0000">Natural history of untreated acute ischemic priapism is (2):</span>'''
*#'''<span style="color:#ff0000">Possible permanent loss of erectile function</span>'''
*#*ED is the most significant complication in patients with prolonged acute ischemic priapism.
*#'''<span style="color:#ff0000">Penile shortening due to corporal fibrosis</span>'''
* '''<span style="color:#ff0000">All patients with persistent acute ischemic priapism should be counseled that there is the chance of erectile dysfunction</span>'''
** '''<span style="color:#ff0000">If acute ischemic priapism event >36 hours, likelihood of erectile function recovery is low.</span>'''
**'''In a patient with acute ischemic priapism >36 hours, surgical interventions, such as distal shunting, with or without tunneling, may be required to achieve detumescence; as it is unlikely the acute ischemic event will resolve with ICI therapy of phenylephrine and aspiration.'''
***'''As the duration of the priapism increases, patients may be refractory to first-line treatments, such as ICI of phenylephrine and aspiration, with or without irrigation.'''


==== Non-surgical management ====
==== Non-surgical management ====
* '''Oral agents are not recommended in the management of acute ischemic priapism'''
* '''Conservative therapies (i.e., observation, oral medications, cold compresses, exercise) are not recommended in the management of acute ischemic priapism'''
* '''<span style="color:#ff0000">Cavernosal aspiration</span>'''
**Minimal corporal blood flow in priapism limits of oral agents
** '''<span style="color:#ff0000">Initial treatment of ischemic priapism is corpora cavernosa decompression by aspiration</span>'''
**Cold compresses should never be used in persons with SCD to avoid provoking vasoconstriction and intravascular sickling
*** '''<span style="color:#ff0000">After performing a dorsal nerve and ring block, a single, large-bore, 19-gauge needle should be inserted at the penoscrotal junction at the 3 or 9 o’clock position to avoid piercing the dorsal neurovascular bundle</span>'''
*'''<span style="color:#ff0000">First-line: intracavernosal phenylephrine AND corporal aspiration, with or without irrigation[https://pubmed.ncbi.nlm.nih.gov/35536142/ ★]</span>'''
*** Aspiration will immediately soften the erection and relieve pain; '''aspiration alone may relieve priapism in 36% of cases'''
**'''<span style="color:#ff0000">Clinicians treating acute ischemic priapism may elect to proceed with alpha adrenergics, or aspiration and saline irrigation, or a combination of both therapies based on their clinical judgment[https://pubmed.ncbi.nlm.nih.gov/35536142/ ★]</span>'''
*** '''Aspiration should be repeated until no more dark blood is coming out from the corpora and fresh bright red blood is obtained'''
***'''<span style="color:#ff0000">ICI with phenylephrine should begin as rapidly as possible following diagnosis[https://pubmed.ncbi.nlm.nih.gov/35536142/ ★]</span>'''
** '''<span style="color:#ff0000">If corporal aspiration is unsuccessful, irrigation with normal saline or injection of an α-adrenergic should follow</span>'''
****These statements are taken near verbatim from AUA guidelines, but are conflicting
* '''<span style="color:#ff0000">Intracavernosal α-adrenergic injection</span>'''
****'''Intracavernosal treatments should not be delayed due to other systemic therapies (e.g., hydration, exchange transfusion), but may be administered concomitantly in most cases.<span style="color:#ff0000">[https://pubmed.ncbi.nlm.nih.gov/35536142/ ★]</span>'''
** '''<span style="color:#ff0000">α-agonists (phenylephrine''', etilefrine, ephedrine, epinephrine, norepinephrine, metaraminol) </span>'''cause cavernous smooth muscle contraction of the cavernous artery and arterioles, and therefore are vasoconstrictors'''
****Even in cases where preserved erectile function is unlikely, clinicians may elect to perform combined treatments to improve penile pain, if present. 
*** '''<span style="color:#ff0000">Potential side effects of intracavernous sympathomimetics:</span>'''
****Intracavernosal therapies may be deferred when ED is anticipated, and expedited placement of a penile prosthesis is planned.
***# '''<span style="color:#ff0000">Headache</span>'''
 
***# '''<span style="color:#ff0000">Dizziness</span>'''
===== Intracavernosal α-adrenergic injection =====
***# '''<span style="color:#ff0000">Hypertension</span>'''
* </span>'''<span style="color:#ff0000">MOA of α-agonists: vasoconstrictors (cause smooth muscle contraction of the cavernous artery and arterioles)'''
***# '''<span style="color:#ff0000">Reflex bradycardia</span>'''
* </span> '''<span style="color:#ff0000">Options: phenylephrine''', etilefrine, ephedrine, epinephrine, norepinephrine, metaraminol)
***# '''<span style="color:#ff0000">Tachycardia</span>'''
**<span style="color:#ff0000">'''Phenylephrine is the'''</span> '''<span style="color:#ff0000">α-agonist of choice in ischemic priapism</span>'''
***# '''<span style="color:#ff0000">Irregular cardiac rhythms</span>'''
***'''Has less systemic side effects compared to other alpha-agonists due to its relatively selectivity for α1-adrenergic receptors with minimal β-mediated ionotropic and chronotropic cardiac effects'''
**** '''<span style="color:#ff0000">Serial monitoring of blood pressure and pulse should be performed during and immediately after intracavernosal injection of sympathomimetic drugs</span>'''
***Use in this context is off-label
*** '''<span style="color:#ff0000">Contraindications (1):</span>'''
 
**** '''<span style="color:#ff0000">Use of monoamine oxidase inhibitor</span>'''
====== Technique<span style="color:#ff0000">[https://pubmed.ncbi.nlm.nih.gov/35536142/ ★]</span> ======
*** '''<span style="color:#ff0000">Phenylephrine</span>'''
*The optimal regimen for phenylephrine dosing, frequency, and method of administration has not been clearly defined
**** '''<span style="color:#ff0000">α-agonist of choice</span>'''
** '''<span style="color:#ff0000">Phenylephrine diluted in normal saline to a concentration of 100-500 mcg/mL</span>''' (0.1-0.5 mg/mL) (optimally premixed by pharmacy to minimize risks of miscalculation/overdose)
**** '''Relatively selective α1-adrenergic receptor agonist with minimal β-mediated ionotropic and chronotropic cardiac effects'''
***Dilute 10mg/mL vial of phenylephrine with 19mL of normal saline to get 10mg/20mL volume = 0.5mg/mL = 500mcg/mL
**** '''Typically diluted in normal saline to a concentration of 100-200 μg/mL and administered intracavernously as a 1-mL injection every 3-5 minutes. The penis is aspirated between successive injections by tightly pinching the shaft at the penoscrotal junction, just below the site of needle insertion'''
** '''Administered intracavernosally (not subcutaneously)'''
**** '''<span style="color:#ff0000">Most common side effects are hypertension and reflex bradycardia</span>'''
** '''Administered laterally (3 or 9 o’clock position) near the base of the penile shaft'''
***** '''<span style="color:#ff0000">No recommendations can be made about maximum safe dosage. Hypertensive stroke has been reported as a complication of cumulative administration of 2 mg</span> [20mL if 100 μg/mL; 10ml if 200 μg/mL]'''
*** Small needles may be used (e.g., 27G) if not aspirating
**** '''The response to phenylephrine decreases with increased duration of priapism'''
** '''<span style="color:#ff0000">Doses administered ≥5 minutes apart'''
***** Corpus cavernosum specimens from patients with prolonged priapism show no contractions to high-dose phenylephrine.
**'''<span style="color:#ff0000">May be continued for up to 1 hour'''
*** '''<span style="color:#ff0000">If the erection persists despite repeated attempts with injections and aspiration/irrigation > 1 hour,  proceed with more definitive therapy (i.e., shunting procedure).'''  
** '''In cases where the combination of phenylephrine and aspiration/irrigation are performed, aspiration should precede phenylephrine administration to permit fresh, oxygenated blood to fill the corpora and potentially improve the yield of phenylephrine administration'''
***'''The penis is aspirated between successive injections by tightly pinching the shaft at the penoscrotal junction, just below the site of needle insertion'''
** '''<span style="color:#ff0000">No recommendations can be made about maximum safe dosage.</span>'''  
***'''Although there is no upper limit to the number of injections which may be performed, injections should be stopped if blood pressure changes are detected.'''
****'''<span style="color:#ff0000">Hypertensive stroke has been reported as a complication of cumulative administration of 2 mg</span> [20mL if 100 μg/mL; 10ml if 200 μg/mL]'''
 
====== <span style="color:#ff0000">Adverse effects of intracavernous sympathomimetics</span> ======
# '''<span style="color:#ff0000">Hypertension (most common)</span>'''
# '''<span style="color:#ff0000">Reflex bradycardia (most common)</span>'''
# '''<span style="color:#ff0000">Tachycardia</span>'''
# '''<span style="color:#ff0000">Irregular cardiac rhythms</span>'''
#'''<span style="color:#ff0000">Headache</span>'''
# '''<span style="color:#ff0000">Dizziness</span>'''
 
* '''Patients receiving intracavernosal injections with phenylephrine should be monitored for blood pressure and heart rate<span style="color:#ff0000">[https://pubmed.ncbi.nlm.nih.gov/35536142/ ★]</span>'''
** Blood pressure and heart rate monitoring seems especially prudent in patients with a history of cardiovascular disease, hypertension, prior stroke, and those using medications such as monoamine oxidase inhibitors (MAOIs).
 
====== <span style="color:#ff0000">Contraindications</span> ======
 
* '''<span style="color:#ff0000">Relative: Use in patients on Monoamine Oxidate Inhibitors (MAOIs) (e.g. isocarboxazid, phenelzine, selegiline, tranylcypromine)</span>'''
** '''No reports of toxicity when used for priapism in males using MAOI'''
**Potentiation of phenylephrine effects by prior administration of MAOI is most significant with use of oral phenylephrine, which is dissimilar from intracavernosal administration.
**Gradual dose escalation may be reasonable when treating priapism in men using these medications.
 
===== Corporal aspiration, with or without irrigation =====
* '''<span style="color:#ff0000">Corporal aspiration'''
**'''<span style="color:#ff0000">Refers to the intracavernosal placement of a needle followed by withdrawal of corporal blood.'''
**Will immediately soften the erection and relieve pain
**May relieve priapism in 36% of cases
*'''<span style="color:#ff0000">Corporal irrigation'''
**'''<span style="color:#ff0000">Refers to subsequent instillation of fluid (typically saline) into the corpora after corporal aspiration.'''
*'''Corporal aspiration and irrigation are often combined''' to remove clotted, deoxygenated blood and restore arterial flow and smooth muscle and endothelial function. 
**They may be performed alone or combined with instillations of phenylephrine.
***No studies have compared aspiration and irrigation with saline to alpha adrenergic injections alone
***Studies suggest greater resolution rates with combination therapy of aspiration, irrigation, and intracavernosal alpha adrenergics compared to alpha adrenergics alone
*'''<span style="color:#ff0000">Technique[https://pubmed.ncbi.nlm.nih.gov/35536142/ ★]</span>'''
** '''Steps for aspiration/irrigation with phenylephrine administration:'''
**# Perform a penile block with local numbing medication (if not previously performed).
**# Place a 16-18 gauge butterfly needle in the 3 or 9 o’clock position (to avoid the dorsal neurovascular bundle) on the penis near the base.
**# Connect the butterfly needle to a 30-60 cc Luer Lock syringe.
**# Alternate between aspiration of blood clots and instillation of saline (chilled if available and if the patient does not have sickle cell disease) until some degree of detumescence can be achieved.
**# Instill phenylephrine.
**# Allow 3-5 minutes of time to pass.
**# Repeat steps 4-6 until detumescence is achieved or until the decision has been made to proceed with surgical shunting.
**# If temporary detumescence is achieved with aspiration followed by a rapid refilling of blood despite multiple attempts of phenylephrine instillation, consideration may be given to placement of a firm penile wrap at the time of aspiration to maintain detumescence.
 
==== Surgical management (Shunting) ====
* '''Principle of shunt procedure is to reestablish corporal inflow by relieving venous outflow obstruction'''; this requires creation of a fistula between the corpora cavernosa and the glans penis, corpora cavernosa and corpus spongiousum, or corpora cavernosa and dorsal or saphenous veins.
 
===== Indications (3)</span> =====
# '''<span style="color:#ff0000">Persistent acute ischemic priapism after intracavernosal phenylephrine and aspiration, with or without irrigation</span>[https://pubmed.ncbi.nlm.nih.gov/35536142/ ★]'''
#*'''Surgical shunting should not be performed until BOTH alpha adrenergics and aspiration and saline irrigation have been attempted</span>[https://pubmed.ncbi.nlm.nih.gov/35536142/ ★]'''
#'''<span style="color:#ff0000">Injections of sympathomimetics has resulted in a significant cardiovascular side effect</span>'''
#'''<span style="color:#ff0000">Malignant or poorly controlled hypertension</span>'''
#
 
* '''<span style="color:#ff0000">Consider for ischemic priapism events ≤72 hours</span>'''
** '''<span style="color:#ff0000">In priapism lasting > 72 hours, consideration should be given to foregoing a shunt</span>'''
 
===== <span style="color:#ff0000">Approach</span> =====
* '''<span style="color:#ff0000">Classification: distal vs. proximal</span>'''
*'''<span style="color:#ff0000">Distal (6)</span>'''
** '''<span style="color:#ff0000">Percutaneous (3)</span>''' (through distal glans towards corpus cavernosum) '''<span style="color:#0000ff">WET'''
**# '''<span style="color:#0000ff">W</span><span style="color:#ff0000">inter: large-bore needle or angiocatheter (least effective)</span>'''
**# '''<span style="color:#0000ff">E</span><span style="color:#ff0000">bbehoj: straight incision''' '''with No. 11 blade'''
**# '''<span style="color:#0000ff">T</span><span style="color:#ff0000"> shunt: No. 10 blade is rotated 90° after insertion'''
**#* '''After Ebbehoj or T shunt, the glans is sutured closed with absorbable suture.''' Discharge home if the penis remains flaccid for 15 minutes. If erection returns or persists, a second T shunt is recommended on the opposite side of the meatus.
** '''<span style="color:#ff0000">Open (3): </span><span style="color:#0000ff">ATC</span>'''
**# '''<span style="color:#0000ff">A</span><span style="color:#ff0000">l -Ghorab</span>: excision of a''' 5-mm '''circular cone segment of the distal tunica albuginea (see Figure in [https://pubmed.ncbi.nlm.nih.gov/27436080/ article])'''
**# '''<span style="color:#0000ff">T</span><span style="color:#ff0000">unnelling</span>''': modification of the Al-Ghorab; '''after excising the circular core of distal tunica albuginea, a 7/8 Hegar dilator is inserted down each corporal body through the tunica window'''
**#'''<span style="color:#0000ff">C</span><span style="color:#ff0000">ombined distal shunt and corporal tunneling</span> <span style="color:#ff0000">maneuver</span>'''
**#*Addition of tunneling may afford slightly higher rates of successful detumescence, but associated with greater degradation of post-procedure erectile function compared to distal shunting alone'''<span style="color:#ff0000">[https://pubmed.ncbi.nlm.nih.gov/35536142/ ★]</span>'''
* '''Proximal (3)'''
**'''Options (open)'''
**# '''Proximal corpus cavernosum to spongiosum shunt (Quackles)'''; require a trans-scrotal or transperineal approach
**# '''Proximal corpus cavernosum to saphenous vein shunt (Grayhack)''' - a wedge of tunica albuginea is removed and the vein is anastomosed end to side of corpora cavernosa.
**#'''Proximal corpus cavernosum to deep dorsal vein shunt (Barry)'''
**'''Represents a historical procedure and has largely been replaced by distal shunts with tunneling procedures<span style="color:#ff0000">[https://pubmed.ncbi.nlm.nih.gov/35536142/ ★]</span>'''
***Technically more difficult to perform that distal, likely no surgeons who have extensive experience<span style="color:#ff0000">[https://pubmed.ncbi.nlm.nih.gov/35536142/ '''★''']</span>
**Should not be considered a mandatory procedure for men who have been confirmed to have failed distal shunting<span style="color:#ff0000">[https://pubmed.ncbi.nlm.nih.gov/35536142/ '''★''']</span>
*'''<span style="color:#ff0000">A distal corporoglanular shunt, with or without tunneling, should be performed in patients with persistent acute ischemic priapism</span>'''
**The optimal type of distal corporoglanular shunt has not been defined'''<span style="color:#ff0000">[https://pubmed.ncbi.nlm.nih.gov/35536142/ ★]</span>'''
**'''<span style="color:#ff0000">Percutaneous distal shunting is less invasive than open distal shunting</span>''' and can be performed with local anesthetic in the emergency department.
** <span style="color:#ff0000">'''Open shunting procedures, especially those that require passage of dilators into the corpora cavernosa, will require general anesthesia and an operating room'''
*'''Inadequate evidence to quantify the benefit of performing a proximal shunt (of any kind) in a patient with persistent acute ischemic priapism after distal shunting.<span style="color:#ff0000">[https://pubmed.ncbi.nlm.nih.gov/35536142/ ★]</span>'''
 
===== Adverse Events of Shunting (6): =====
#'''<span style="color:#ff0000">Penile edema'''
#'''<span style="color:#ff0000">Hematoma'''
#'''<span style="color:#ff0000">Infection'''
#'''<span style="color:#ff0000">Urethral fistula'''
#'''<span style="color:#ff0000">Penile necrosis'''
#'''<span style="color:#ff0000">Pulmonary embolism'''
 
===== Methods to prevent shunt obstruction and subsequent failure (3): =====
# Avoid compressive penile dressings
# Consider anticoagulation
# Patient should periodically squeeze and release the distal penis to “milk” the shunt maintaining patency
 
==== Resolution of acute ischemic priapism ====
*'''<span style="color:#ff0000">Characterized by the penis returning to a flaccid, nonpainful state, with restoration of penile blood flow.</span>'''
**'''Oftentimes, persistent penile edema, ecchymosis, and partial erections occur and mimic unresolved priapism.'''
***This often relates to the duration of priapism and may also signify segmental regions of cavernosal ischemia/necrosis.
*'''<span style="color:#ff0000">After shunting, follow-up with the patient regarding erectile function and any subsequent ED therapies</span>'''
 
==== Special Scenarios ====
 
===== Priapism refractory to shunting =====
*'''<span style="color:#ff0000">A vascular study (such as a PDUS) or cavernosal blood gas should be performed prior to performing additional interventions (repeat distal or proceeding to proximal shunting).</span>[https://pubmed.ncbi.nlm.nih.gov/35536142/ ★]'''
**Goal is to assess penile hemodynamic characteristics and extent of necrosis/fibrosis to differentiate persistent acute ischemic priapism from reactive hyperemia or conversion to non-ischemic priapism and inform secondary treatment decisions
***Penile corporal blood gas can help with decision making about proceeding to additional surgical procedures including placement of an immediate penile prosthesis.
***Penile duplex ultrasound
****Can be used to
*****Differentiate between acute ischemic and non-ischemic priapism
*****Determine shunt patency by showing restoration of cavernosal arterial inflow in a patient who has undergone a distal shunt
**Further surgical decisions should not be based only on exam
**'''Evaluating the status of a patient with refractory priapism is particularly important in the event that a patient is referred from another institution and/or the clinician is seeing a patient who had been previously treated elsewhere and a complete patient history may not be available.'''
 
===== Acute Ischemic Priapism >36 hours =====
*'''<span style="color:#ff0000">Options</span>[https://pubmed.ncbi.nlm.nih.gov/35536142/ ★]'''
*#'''<span style="color:#ff0000">Observation, pain control, and outpatient follow-up</span>'''
*#*May bypass more invasive procedures (e.g., surgical shunting).
*#'''<span style="color:#ff0000">Distal shunting, with or without tunneling</span>'''
*#'''<span style="color:#ff0000">Early implantation of penile prosthesis</span>'''
*'''<span style="color:#ff0000">Unlikely the acute ischemic event will resolve with ICI therapy of phenylephrine and aspiration.</span>[https://pubmed.ncbi.nlm.nih.gov/35536142/ ★]'''
**'''The response to phenylephrine decreases with increased duration of priapism'''
 
*'''It is important to note that before considering conservative management or penile prosthesis placement in men with a priapism >36 hours, the timeline should be sufficiently confirmed.'''
**Patient histories relating to an exact timeline may often be unreliable, particularly in cases of concomitant substance use, episodes of intermittent detumescence, recurrent priapism (e.g., SCD), or partial (not fully rigid) erections. In these settings, clinical judgment is required to identify the true timeline for onset of ischemia (i.e., onset of severe, persistent penile pain). 
**If the timeline is in question, clinicians should preferentially attempt to decompress the priapism, particularly in younger men or those with high baseline erectile function.
 
===== Early implantation (within 2 weeks) of penile prosthesis =====
* '''<span style="color:#ff0000">Indications</span>'''
**'''<span style="color:#ff0000">May be considered in a patient with untreated acute ischemic priapism > 36 hours or in those who are refractory to shunting, with or without tunneling. </span>[https://pubmed.ncbi.nlm.nih.gov/35536142/ ★]'''
*'''Advantages to prostheses placed in the setting of acute ischemic priapism (5):<span style="color:#ff0000">[https://pubmed.ncbi.nlm.nih.gov/35536142/ ★]</span>'''
*# '''Detumescence'''
*# '''Relief of pain'''
*#'''Preservation of penile length'''
*#'''Return to sexual activity'''
*#'''Overall satisfaction'''
* '''Disadvantages to immediate implantation (3):'''
*# '''Urologist involved for may lack the experience, comfort level, or materials to render device placement practical and/or possible<span style="color:#ff0000">[https://pubmed.ncbi.nlm.nih.gov/35536142/ ★]</span>'''
*#Repetitive bedside irrigation procedures may, in theory, increase the chances for bacterial entry into the corpora that could threaten an implant with infection.
*##'''Infection rates''' < 10%, '''higher with delayed implantation<span style="color:#ff0000">[https://pubmed.ncbi.nlm.nih.gov/35536142/ ,★]</span>'''
*#'''Distal shunts may have compromised the integrity of the tunica albuginea that would surround an implant, possibly predisposing to erosion.'''
 
===== Recurrent Ischemic Priapism =====
* '''Options''' '''to prevent subsequent episodes<span style="color:#ff0000">[https://pubmed.ncbi.nlm.nih.gov/35536142/ ★]</span>'''
**'''Idiopathic (7):'''
**#'''<span style="color:#ff0000">Ketoconazole with prednisone'''
**#*'''Highest success rate'''
**#*'''<span style="color:#ff0000">Adverse effects'''
**#*#'''<span style="color:#ff0000">Liver toxicity, thus warranting frequent assessment of liver function tests</span>'''
**#'''<span style="color:#ff0000">Pseudoephedrine</span>''', an oral α-adrenergic agonist, promotes muscle contraction within the erectile tissue
**#'''<span style="color:#ff0000">Phosphodiesterase type 5 inhibitors</span>'''
**#'''Aspirin'''
**#'''Baclofen'''
**#'''Dutasteride'''
**#'''Cyproterone acetate'''
**##Anti-androgen
**##Not available in the United States
**'''Associated with sickle cell disease'''
**#'''Same options as idiopathic'''
**#'''Etilefrine'''
**#'''Hydroxyurea'''
**#'''Automated exchange transfusion'''
**'''Drug therapy is typically initiated at bedtime'''
**Patients with recurrent ischemic priapism should be informed that hormonal regulators (ketoconazole, cyproterone acetate) may impair fertility and sexual function
***Adverse effects, due to manipulation of hypothalamic-pituitary-gonadal axis
****Fatigue
****Hot flashes
****Breast tenderness
****Changes in mood
****ED
****Negatively impact sperm parameters
**'''Home self-injection of intracavernous α-adrenergic agent'''
*** Not a preventative strategy; may avert a full-blown episode of ischemic priapism when administered at home for prolonged morning erections
**'''Optimal strategy is unknown'''


==== Surgical management ====
===== Sickle Cell Disease and other Hematologic Disorders =====
* '''<span style="color:#ff0000">Indications (2):</span>'''
* '''<span style="color:#ff0000">The best intervention is to relieve episodes with prompt intracavernosal phenylephrine and corporal aspiration, with or without irrigation, as in other acute ischemic priapism patients, before proceeding to systemic therapies specific to the underlying disorder</span>[https://pubmed.ncbi.nlm.nih.gov/35536142/ ★]'''
*# '''<span style="color:#ff0000">Failure of repeated penile aspirations and injections of sympathomimetics</span>'''
*'''Standard sickle cell assessment and interventions should be considered concurrent with initiation of urologic intervention. Specifically, disease specific systemic care should address:'''
*# '''<span style="color:#ff0000">Injections of sympathomimetics has resulted in a significant cardiovascular side effect</span>'''
** '''Hydration with IV fluid only if made NPO (maintenance rate) or dehydrated (replace deficit plus maintenance rate)'''
* '''Early surgical intervention may be preferable in patients with:'''
*** '''Hyperhydration is not indicated and may predispose to acute chest syndrome.'''
*# '''Malignant or poorly controlled hypertension'''
** '''Supplemental oxygenation only if hypoxic.'''
*# '''Using monoamine oxidase inhibitor medications contraindicating α-adrenergic therapies'''
** '''Pain management with oral or parenteral opioids as per usual painful events (remembering that some patients with SCD may be tolerant to analgesia because of those prior experiences).'''
* '''<span style="color:#ff0000">Shunting</span>'''
** '''Hematologic status comparison of CBC and reticulocyte count to baseline values'''
** '''Principle of shunt procedure is to reestablish corporal inflow by relieving venous outflow obstruction'''; this requires creation of a fistula between the corpora cavernosa and the glans penis, corpora cavernosa and corpus spongiousum, or corpora cavernosa and dorsal or saphenous veins.
*** '''Best done in consultation with the patient’s hematologist.'''  
** '''<span style="color:#ff0000">Consider for ischemic priapism events ≤72 hours</span>'''
*** '''Transfusion is not indicated if hemoglobin is near usual value, and over-transfusion may be associated with neurologic events.'''
*** '''<span style="color:#ff0000">In priapism lasting > 72 hours, consideration should be given to foregoing a shunt</span>'''
*** '''Acute exchange transfusion is not indicated.'''
** '''<span style="color:#ff0000">Classified: distal vs. proximal</span>'''
*** If operative shunting procedures are required, consideration should be given to a simple transfusion of packed red blood cells to raise the hemoglobin to 9-10 g/dl prior to general anesthesia
*** '''<span style="color:#ff0000">Distal (6)</span>'''
*** Rarely are blood products required before an aspiration and irrigation procedure, the one exception may be with a very low platelet count (<20,000/uL).
**** '''<span style="color:#ff0000">Percutaneous (3)</span>''' (through distal glans towards corpus cavernosum) '''<span style="color:#0000ff">WET'''
** '''Presence of other acute sickle cell events: neurologic disorders including acute stroke, acute chest syndrome, biliary colic, renal insufficiency which while not associated with a higher frequency of priapism may present at the same time.'''
****# '''<span style="color:#0000ff">W</span><span style="color:#ff0000">inter: large-bore needle or angiocatheter'''
* '''Ice packs and other cold compresses should never be used in SCD patients as they may worsen painful events by precipitating intravascular sickling.'''
****# '''<span style="color:#0000ff">E</span><span style="color:#ff0000">bbehoj: straight incision''' '''with No. 11 blade'''
*Most patients with SCD experience recurrent short ischemic priapism events, (lasting <4 hours and commonly referred to as stuttering priapism) but acute episodes and particularly recurrent acute episodes occur commonly enough (both before and after shorter, stuttering events) that education about when to seek urologic attention is a critical part of the patient education in SCD disorders.
****# '''<span style="color:#0000ff">T</span><span style="color:#ff0000"> shunt:''' '''No. 10''' '''blade is rotated 90° after insertion'''
*Drugs that have been tried to prevent subsequent priapism episodes
****#* '''After Ebbehoj or T shunt,the glans is sutured closed with absorbable suture.''' Discharge home if the penis remains flaccid for 15 minutes. If erection returns or persists, a second T shunt is recommended on the opposite side of the meatus.
**Etilefrine
**** '''<span style="color:#ff0000">Open (3): </span><span style="color:#0000ff">ACC</span>'''
**Ephedrine
****# '''<span style="color:#0000ff">A</span><span style="color:#ff0000">l -Ghorab</span>: excision of a''' 5-mm '''circular cone segment of the distal tunica albuginea'''
**Pseudoephedrine
****# '''<span style="color:#0000ff">C</span><span style="color:#ff0000">orporal snake</span>''': modification of the Al-Ghorab; '''after excising the circular core of distal tunica albuginea, a 7/8 Hegar dilator is inserted down each corporal body through the tunica window'''
**Terbutaline
****# '''<span style="color:#0000ff">C</span><span style="color:#ff0000">ombined T shunt and corporal snake maneuver</span>'''
**PDE5is (e.g., sildenafil, tadalafil)
*** '''<span style="color:#ff0000">Proximal (2) (open)</span>'''
**5 alpha reductase inhibitors (dutasteride or finaseride)
***# '''<span style="color:#ff0000">Proximal corpus cavernosum to spongiosum shunt (Quackles)</span>'''; require a trans-scrotal or transperineal approach
**Anti-androgens (cyproterone, bicalutamide, leuprolide, stilboesterol)
***# '''<span style="color:#ff0000">Proximal corpus cavernosum to saphenous vein or deep dorsal vein shunt</span>''' - a wedge of tunica albuginea is removed and the vein is anastomosed end to side of corpora cavernosa.
**Ketoconazole/prednisone
*** '''<span style="color:#ff0000">Distal shunts are preferred over proximal because they are technically easier to perform</span>'''
**Hydroxyurea
**** '''<span style="color:#ff0000">Percutaneous distal shunting is less invasive than open distal shunting</span>''' and can be performed with local anesthetic in the emergency department.
*Ongoing chronic (monthly) exchange transfusions do appear to be associated with a reduction in acute and stuttering priapism episodes.
**** <span style="color:#ff0000">'''Open shunting procedures, especially those that require passage of dilators into the corpora cavernosa, will require general anesthesia and an operating room'''
**Exchange transfusion should not be as the primary treatment in patients with acute ischemic priapism associated with sickle cell disease.
**** '''<span style="color:#ff0000">Consider open distal shunting if percutaneous distal shunting fails. If distal shunting fails, then proximal shunting is recommended</span>'''
**For prolonged acute priapism events that cannot be relieved with intracavernosal phenylephrine and corporal aspiration, exchange transfusion can be considered
*** '''When ischemic priapism has been present for > 36 hours, immediate placement of bilateral T shunts is recommended''', '''with passage of 20-Fr dilators into the fistula tract''' and well into the corpora cavernosa down to the crus. This technique is more traumatic and '''will require general anesthesia'''
*
** '''Complications of shunting (6):'''
**#'''Penile edema'''
**#'''Hematoma'''
**#'''Infection'''
**#'''Urethral fistula'''
**#'''Penile necrosis'''
**#'''Pulmonary embolism'''
** '''Methods to prevent shunt obstruction and subsequent failure (3):'''
**# Avoid compressive penile dressings
**# Consider anticoagulation
**# Patient should periodically squeeze and release the distal penis to “milk” the shunt maintaining patency
** '''After shunting, follow-up with the patient regarding erectile function and any subsequent ED therapies'''
* '''Immediate implantation of penile prosthesis'''
** Some have suggested performing an immediate penile prosthesis procedure in the acute management of ischemic priapism >48 hours in patients in whom aspiration, irrigation, phenylephrine, and shunting have failed.
** '''Advantages to immediate implantation (2):'''
**# '''Corporal fibrosis not yet established'''
**# '''Penile length may be preserved'''
** '''Disadvantage to immediate implantation (1):'''
**# '''Significantly higher rates of complications''' (infection, urethral injury, device migration, device erosion, and revision surgeries) when inserted in the acute managment of ischemic priapism compared to elective insertion in a patient with erectile dysfunction


==== Sickle cell disease induced ischemic priapism ====
===== Prolonged Erection Following Intracavernosal Vasoactive Medication =====
* '''Classically, treatment of SCD-induced ischemic priapism involved analgesics, hydration, oxygen, bicarbonate, and exchange transfusion. Unfortunately, acute neurologic complications may follow exchange transfusions. Hematologists have begun to question the emphasis on intravenous hydration, sodium bicarbonate for alkalinization, and exchange transfusion as first line therapy for SCD-associated priapism; hematologic consultation should be obtained in the management of priapism but hematologic therapy alone is not effective management of SCD priapism'''
* ''Initial treatment of low-flow priapism resulting from sickle cell disease is conservative, with hydration, oxygenation, alkalization, analgesia, and exchange transfusion aimed at reducing HbS concentration (from Chapter 146)''
* ''Evacuation of blood and irrigation of the corpora cavernosa along with intracavernous injections of α-adrenergic sympathomimetic agents, such as phenylephrine or epinephrine solution, can be a concurrent therapy (from Chapter 146)''


=== Stuttering priapism ===
* '''<span style="color:#ff0000">A prolonged erection (≤4 hours in duration) following iatrogenic- or patient self-administration of erectogenic medications into the corpus cavernosum (ICI) represents a distinct pathology when compared to acute ischemic priapism or non-ischemic priapism.</span>'''
* '''Drug therapy is typically initiated at bedtime'''
**Much more common and may be managed differently than acute ischemic priapism
* '''Options (5):'''
*'''<span style="color:#ff0000">Management</span>'''
*# '''Pseudoephedrine, an oral α-adrenergic agonist, promotes muscle contraction within the erectile tissue'''
**See [https://www.auanet.org/documents/Guidelines/PDF/priapism/NIP%20JU%20SUMMARY%20Figure%20Three%20Prolonged%20Erections.pdf AUA/SMSNA Guideline Flowchart on Management of Prolonged Erection]
*# '''If pseudoephedrine is unsuccessful, other agents can be used, including an oral β agonist'''
**'''If erection > 4 hours, treat according to ischemic priapism algorithm'''
*# Recently, preliminary findings have suggested that the use of '''continuous, long-term oral PDE5 inhibitor therapy''' may prevent recurrent priapism based on the hypothesis that PDE5 dysregulation may be involved in priapism
***'''All patients should be instructed at the time of ICI training, or after receiving an in-office erectogenic therapy, that they should return to either the office or emergency department if they experience an erection lasting > 4 hours.<span style="color:#ff0000">[https://pubmed.ncbi.nlm.nih.gov/35536142/ ★]</span>'''
*# '''GnRH agonists or anti-androgens''' (e.g. bicalutamide)
**'''<span style="color:#ff0000">If prolonged erection 1-4 hours after home ICI or following an in-office injection</span>'''
*#* Can be used to suppress the androgenic effects on penile erection.
***'''<span style="color:#ff0000">Options</span>'''
*#* '''Chronic administration may affect libido or fertility,''' cause gynecomastia, cause hot flushes, promote osteoporosis, increase the risks of cardiovascular disease, and worsen sexual function
***#'''<span style="color:#ff0000">Conservative options (in the case of home ICI)</span>''' including ice compresses to the penis, laying supine, ejaculation, exercise, penile compresses and oral medications such as pseudoephedrine, in the absence of any clinical data demonstrating efficacy
*#** May be used in adults but should not be used in patients who have not achieved full sexual maturation and adult stature.
***#*Utilize intracavernosal phenylephrine if conservative management is ineffective in the treatment of a prolonged erection.
*# '''Intracavernous α-adrenergic agent'''
***#'''<span style="color:#ff0000">In-office phenylephrine</span>'''
*#* May avert a full-blown episode of ischemic priapism when administered at home for prolonged morning erections
***#* '''Not appropriate for clinicians who administer in-office erectogenic medications to refer the patient to the emergency department as a matter of routine following an in-office injection, rather, the patient should return to the office for detumescence whenever possible.'''
*#* Intracavernosal injection of phenylephrine (by the adult patient or parent) '''should be considered as an adjunct to daily systemic therapies in patients with stuttering priapism'''
***#*'''Intracavernosal aspiration and irrigation likely represents too aggressive of a therapy for this specific clinical scenario to be used as a first-line therapy'''
***#** Intracavernosal phenylephrine can be delivered through a small needle (27G) compared to needle typically used for aspiration/irrigation (16 or 18G)
***#**The physiologic rationale for aspiration and irrigation is to remove intracavernosal clots and permit entry of fresh blood in an attempt to restore smooth muscle function and vascular drainage.  As the pathologic state of intracavernosal clotting and ischemia likely is not present with prolonged erections <4 hours, aspiration and irrigation is rarely warranted.  
***#*** Persistent, prolonged erections may be considered for aspiration and irrigation if phenylephrine alone is unsuccessful.
***'''Duration of a persistent erection requiring intervention is not clearly defined. Generally,'''
**** '''Erection lasting'''
***** '''<1 hour post injection would not require intervention'''
***** '''>4 hours would warrant treatment, regardless of underlying etiology'''
***** '''Decision to intervene in the time-period between 1 and 4 hours would depend on several clinical factors:'''
*****# '''Penile rigidity'''
*****#* Mild erection (i.e., not sufficient to penetrate without assistance) would not require treatment, whereas a fully rigid erection might
*****#* Intermittently rigid erection is considered differently than a fully rigid erection, which has remained persistent since the original injection
*****# '''Specific medication used and dosage'''
*****#* Alprostadil alone is likely associated with shorter durations of erections and likely has a lower risk of ischemic priapism compared to combination therapies, which include  papaverine and/or phentolamine
*****#* Higher dosages are empirically more likely to result in a prolonged erection compared to lower ones
*****# '''Age'''
*****# '''Baseline erectile function'''
*****# '''Reliability/capacity'''
*****# '''Comorbid conditions'''
*****# Pain as an indicator for treatment is not relevant in many scenarios, as the intracavernosal medications themselves are often associated with penile pain


=== Non-ischemic priapism ===
=== Non-ischemic priapism ===
* '''Cavernous aspiration has only a diagnostic role in nonischemic priapism'''.
* See [https://www.auanet.org/documents/Guidelines/PDF/priapism/NIP%20JU%20SUMMARY%20Figure%20Four%20Treatment%20of%20Non-Ischemic%20Priapism.pdf AUA/SMSNA Guideline Flowchart on Management of Non-ischemic Priapism]
*'''Cavernous aspiration has only a diagnostic role in nonischemic priapism'''.
** Repeated aspirations, injection, and irrigation with intracavernous sympathomimetics have no role in the treatment of nonischemic priapism.
** Repeated aspirations, injection, and irrigation with intracavernous sympathomimetics have no role in the treatment of nonischemic priapism.
* '''First-line: observation'''
* '''<span style="color:#ff0000">First-line: observation</span>'''
** '''Non-ischemic priapism is not an emergency; initial observation is recommended'''
** '''<span style="color:#ff0000">Non-ischemic priapism is not an emergency; initial observation is recommended</span>[https://pubmed.ncbi.nlm.nih.gov/35536142/ ★]'''
** '''Spontaneous resolution or response to conservative therapy has been reported in up to 62% of cases'''
** '''Spontaneous resolution or response to conservative therapy has been reported in up to 62% of cases'''
*** No comparative studies of intervention vs. conservative management in non-ischemic priapism
*** No comparative studies of intervention vs. conservative management in non-ischemic priapism
** Conservative measures include ice applied to the perineum and site-specific compression
** Conservative measures include ice applied to the perineum and site-specific compression
* '''Second-line: embolization with interventional angiography'''
**'''<span style="color:#ff0000">4-week period is reasonable, unless the patient is severely bothered by the tumesced penis</span>[https://pubmed.ncbi.nlm.nih.gov/35536142/ ★]'''
** '''Patients demanding immediate relief can be offered selective arterial embolization'''
***After the 4-week mark, the patient’s fistula can be re-evaluated using penile duplex ultrasound; the patient’s sexual function and degree of bother can be further quantified. In cases where the fistula is unchanged and/or where patient bother is significant, intervention may be considered.
*** '''Although ultimately successful, embolization of non-ischemic priapism may require retreatment;''' '''a single treatment of embolization carries a recurrence rate of 30%.'''
**'''Consider penile duplex ultrasound for assessment of fistula location and size in a patient with diagnosed non-ischemic priapism'''
*** '''Normal post-embolization erectile function has been reported in 75-86% of patients. Bilateral arterial embolization significantly increased the risk of ED.'''
***Screening for anatomical abnormalities and identification of cavernous artery fistula (turbulent flow may be detected) or pseudoaneurysm location and size
***Ultrasonography is of particular benefit in a patient with NIP being considered for fistula embolization. T
****Allows for communication between the urologist and radiologist prior to intervention regarding fistula location, size, and eventual choice of vascular access.
***Ultrasonography may also potentially help with the follow-up of a patient with NIP opting for observation through tracking of fistula and its size.
* '''<span style="color:#ff0000">Second-line: percutaneous fistula embolization</span>'''
** '''<span style="color:#ff0000">Indications</span>'''
***'''<span style="color:#ff0000">Persistent non-ischemic priapism who have failed a period of observation and are bothered by persistent penile tumescence, and who wish to be treated</span>'''
**'''<span style="color:#ff0000">Prior to embolization</span>'''
***'''<span style="color:#ff0000">Fistula should be readily visible on a PDUS.</span>'''
***'''<span style="color:#ff0000">Patients should be informed that embolization carries a risk of erectile dysfunction, recurrence, and failure to correct non-ischemic priapism.</span>'''
****Pooled analysis suggest that embolization resulted in penile detumescence in 85% of patients, with 80% of men retaining functional erections
*****'''Bilateral arterial embolization significantly increased the risk of ED.'''
**Embolization should only be attempted by an experienced interventional radiologist.
** '''<span style="color:#ff0000">In patients who have failed an initial attempt at embolization, patients should be offered a second attempt at an embolization procedure</span>[https://pubmed.ncbi.nlm.nih.gov/35536142/ ★]'''
***Embolization of non-ischemic priapism may require retreatment; a single treatment of embolization carries a recurrence rate of 30%.
***Second attempt at an embolization procedure likely to be more effective and safer than an attempt at surgical ligation, given the lack of experience in the latter approach for most urologists and the poor data supporting ligation.
* '''Surgery'''
* '''Surgery'''
** '''In cases of longstanding arterial priapism in which a pseudocapsule around the fistula has developed,''' '''surgical ligation''' has been reported to be successful. The surgical approach is transcorporal.
** Surgical ligation of the corporo-cavernosal fistula following failed attempts at embolization (or when embolization is not available at the center treating the patient) is an option for patients with non-ischemic priapism
** '''Formation of a pseudocapsule may take weeks to months after trauma. Corporal exploration before the formation of a pseudocapsule may result in ligation of the cavernous artery rather than selective ligation of the fistula.'''
**'''The lack of familiarity of most urologists with this surgical approach makes the procedure particularly challenging[https://pubmed.ncbi.nlm.nih.gov/35536142/ ★]'''
** '''Currently this intervention is reserved for patients:'''
***Surgical approach is transcorporal
**# '''Who do not wish to pursue expectant management and are poor candidates/refuse angioembolization'''
**# '''In places where technology is not available'''
**# '''In whom angioembolization has failed'''


== Questions ==
== Questions ==
Line 358: Line 616:


* Wein AJ, Kavoussi LR, Partin AW, Peters CA (eds): CAMPBELL-WALSH UROLOGY, ed 11. Philadelphia, Elsevier, 2015, vol 1, chap 58
* Wein AJ, Kavoussi LR, Partin AW, Peters CA (eds): CAMPBELL-WALSH UROLOGY, ed 11. Philadelphia, Elsevier, 2015, vol 1, chap 58
*[https://pubmed.ncbi.nlm.nih.gov/34495686/ Bivalacqua, Trinity J., et al. "Acute ischemic priapism: an AUA/SMSNA guideline." ''Journal of Urology'' 206.5 (2021): 1114-1121.]

Latest revision as of 15:36, 18 March 2024

Includes 2021 AUA/SMSNA Guidelines on Priapism

  • Guideline search up to February 2021

See Video Review on AUA Guidelines on Priapism

Definition[edit | edit source]

  • Priapism: a persistent penile erection that continues > 4 hours beyond, or is unrelated to, sexual stimulation

Classification (3)[edit | edit source]

  1. Ischemic
  2. Non-ischemic
  3. Recurrent non-ischemic (stuttering)

Ischemic priapism (veno-occlusive, low-flow)[edit | edit source]

  • Majority of cases
  • Characterized by little or no cavernous blood flow
  • Associated with (features that distinguish this from non-ischemic priapism)
    1. Pain
    2. Fully rigid and tender corpora cavernosa
    3. Abnormal blood gas (i.e., hypoxic, hypercarbic, acidotic)
    4. Requires prompt evaluation and may require emergency management
    5. Non-traumatic etiology
  • Natural history of untreated acute ischemic priapism includes days to weeks of painful erections followed by permanent loss of erectile function
  • Duration of ischemic priapism is associated with the risk of future erectile dysfunction

Non-ischemic priapism (arterial, high-flow)[edit | edit source]

  • Relatively rare
  • May last hours to weeks and is frequently recurrent
  • Pathogenesis:
    • Likely results from unregulated control of arterial inflow and cavernous smooth muscle tone
    • Usually involves perineal or penile trauma resulting in laceration of the cavernous artery (or one of its branches within the corpora) and unregulated cavernous arterial inflow.
      • Most common cause is a straddle injury to the crura
        • Other mechanisms of traumatic arterial laceration include coital trauma, kicks to the penis or perineum, pelvic fractures, birth canal trauma to the male newborn, needle lacerations, iatrogenic needle injury, complications of penile diagnostics, vascular erosions complicating metastatic infiltration of the corpora, and after iatrogenic trauma from coldknife urethrotomy, corporoplasty, and penile revascularization procedures
          • Sustained partial erection may develop 24 hours after perineal or penile blunt trauma
      • The arteriolacunar fistula is typically unilateral.
    • After either aggressive medical management of ischemic priapism or surgical shunting, priapism may rapidly recur with conversion from ischemic (low-flow) to non-ischemic (high-flow)
    • Other causes include Fabry disease and sickle cell anemia
  • Associated with (features that distinguish this from ischemic priapism)
    1. Lack of pain (beyond pain from potential acute traumatic etiology)
    2. Tumescent but less rigid corpora cavernosa
      • Erection is partial and bendable because there is no restriction of venous outflow,
      • Patients do report additional engorgement with sexual stimulation, with return to partial erection after climax
    3. Normal penile blood gas (features that distinguish this from non-ischemic priapism)
    4. Once properly diagnosed, does not require emergent intervention
    5. Traumatic etiology, usually

Recurrent ischemic priapism[edit | edit source]

  • Also known as "stuttering" priapism
  • Definition: Recurrent ischemic episodes, with or without meeting the 4-hour time criteria for priapism
    • Unwanted painful erections occur repeatedly with intervening periods of detumescence
    • The key differentiating factor between the current definition of recurrent ischemic priapism and other recurrent priapism-like conditions is the requirement of confirmed penile ischemia.
      • Priapism-like conditions include sleep-related painful erections, undesired prolonged erections, and recurrent non-ischemic priapism
  • Both acute ischemic priapism and non-ischemic priapism may recur over time
  • Commonly associated with sickle cell disease
    • Priapism in children and adolescents is most commonly related to sickle cell disease
      • The sickle cell genetic mutation is the result of a single amino acid substitution in the beta-globin subunit of hemoglobin S.
      • Clinical features seen in homozygous sickle cell disease: chronic hemolysis, vascular occlusion, tissue ischemia, and end-organ damage.
      • Sickle cell trait can be a predisposing factor to ischemic priapism.
    • Pathogenesis is related to hemolysis and reduced nitric oxide
  • Management focuses on
    • Treatment of acute episodes
    • Prevention of future events

Pathogenesis[edit | edit source]

Risk factors (8)[edit | edit source]

  1. Medications (9):
    1. α-blockers (tamsulosin, terazosin, doxazosin, prazosin)
    2. Anti-anxiety agents (hydroxyzine)
    3. Anti-depressants and antipsychotics (trazodone, bupropion, fluoxetine, sertraline, lithium, clozapine, risperidone, olanzapine, chlorpromazine, thioridazine, phenothiazines)
    4. Anti-coagulants (heparin, warfarin)
    5. Anti-hypertensives (hydralazine, propranolol, guanethidine)
    6. Attention-deficit/hyperactivity disorder agents (methylphenidates (concerta, daytrana, focalin, metadate, methylin, quillivant, ritalin), atomoxetine (strattera))
    7. Recreational drugs (alcohol, cocaine (intranasal and topical), marijuana)
    8. Hormones (testosterone, gonadotropin-releasing hormone)
    9. Vasoactive erectile agents (oral phosphodiesterase type 5 (PDE5) inhibitors, papaverine, phentolamine, prostaglandin E1, combination intracavernous therapy)
      • Most case reports describing priapism after use of a PDE5 inhibitor reveal histories of increased risk of priapism: sickle cell disease, spinal cord injury, use of a PDE5 inhibitor recreationally, use of a PDE5 inhibitor in combination with intracavernosal injections, history of penile trauma, use of psychotropic medications, or use of recreational drugs; risk is even lower for daily dosing PDE5 inhibitor
  2. Genitourinary trauma (straddle injury, pelvic trauma, coital injury, kick to penis or perineum)
  3. Genitourinary conditions (urinary retention, arteriovenous or arteriocavernous bypass surgery)
  4. Thrombotic diseases/states (thrombophilc states (deficiencies of protein C, S or Factor V Leiden), asplenia, erythropoietin use, hemodialysis with heparin use, and cessation of warfarin)
  5. Hematologic diseases (sickle cell disease, thalassemia, hemolytic anemias (congenital dyserythropoietic anemia type II, unstable hemoglobinopathies, polycythemia, thrombotic thrombocytopenic purpura, multiple myeloma, chronic myelogenous or lymphocytic leukemias)
    1. In homozygous sickle cell anemia, the most common form of SCD, priapism occurs in 23-89% of males by age 18.[2]
  6. Infectious (toxin-mediated) causes (scorpion sting, spider bite, rabies, malaria)
  7. Metabolic conditions (amyloidosis, Fabry disease, gout)
  8. Cancer (metastatic or regional infiltration) (prostate, urethra, testis, bladder, rectum, lung, kidney)
  9. Neurogenic conditions (spinal cord injury, cerebral vascular accident, brain tumor, syphilis, autonomic neuropathy, lumbar disk herniation, spinal stenosis, spinal anesthesia, cauda equina syndrome)

Diagnosis and Evaluation[edit | edit source]

UrologySchool.com Summary[edit | edit source]

  • Recommended
    • History and Physical Exam
    • Labs (2)
      1. Corporal blood gas
  • Optional
    • Labs (3)
      1. CBC
      2. Hemoglobin electrophoresis
      3. Screening for psychoactive drugs and urine toxicology
    • Imaging
      • Penile duplex Doppler ultrasonography
        • When the diagnosis of acute ischemic versus non-ischemic priapism is indeterminate

History and Physical Exam[edit | edit source]

  • In the majority of cases, the differentiation of acute ischemic priapism versus non-ischemic priapism may be made using only the history and physical exam. 
    • In cases where the subtype is indeterminate, additional testing may be warranted.

History[edit | edit source]

  • Signs and Symptoms
    • Sexual history
      • Duration of erection, presence of pain
      • Baseline erectile function
  • Risk factors
    • Medical history
      • Personal or family history of sickle cell disease, hemoglobinopathies, hypercoagulable states
      • Pelvic, genital, or perineal trauma, especially a perineal straddle injury
      • Previous episodes of priapism and method of treatment
    • Medications that might have precipitated the episode
      • Use of any erectogenic therapies (both prescription and nutritional supplements)
      • Use of recreational drugs

Physical Exam[edit | edit source]

  • Genitalia
    • The corpora cavernosa are typically affected while the corpus spongiosum and the glans penis are not
    • In ischemic priapism, the corpora cavernosa are often fully rigid and tender
    • In non-ischemic priapism, the corporal cavernosa will be tumescent, partially erect, but not completely rigid. The penis is also unlikely to be tender
  • Abdominal, pelvic, and perineum
    • May reveal evidence of trauma or malignancy.

Laboratory[edit | edit source]

  • Corporal blood gas by aspiration
    • Should be obtained in the emergency evaluation of priapism
      • Clinical situations where a blood gas may be omitted at the clinician’s discretion (3):
        1. Priapism induced by in-office or at home ICI therapies
        2. Cases of recurrent ischemic priapism (i.e., SCD)
        3. When the diagnosis is abundantly clear by history and examination alone
    • Most common diagnostic methods of distinguishing acute ischemic priapism from non-ischemic priapism when the diagnosis cannot be made by history alone
    • Blood aspirated from the corpus cavernosum in patients with acute ischemic priapism is hypoxic (dark red), while corporal blood in NIP patients is normally oxygenated (bright red)
Source PO2 (mm Hg) PCO2 (mm Hg) pH
Normal arterial blood (room air) >90 <40 7.40
Normal mixed venous blood (room air) 40 50 7.35
Ischemic priapism (first corporal aspirate) <30 >60 <7.25
  • CBC
    • May identify elevated white blood cell counts, potentially identifying cases where priapism is due to underlying malignancy (e.g., leukemia). 
    • Among men with sickle cell disease, acute ischemic priapism is associated with lower hemoglobin and elevated lactate dehydrogenase, bilirubin, aspartate aminotransferase, reticulocyte count, white blood cells, and platelet counts.
    • Platelet and eosinophil counts may also be elevated in men with acute ischemic priapism.
  • Hemoglobin electrophoresis and other sickle cell testing
    • May be appropriate in select clinical scenarios and based on underlying clinical suspicion (e.g., patient race)
      • In most cases, most men with sickle cell disease have been diagnosed previously. 
        • The yield of identifying men with previously undiagnosed SCD among a cohort of men presenting with priapism is not well established. 
  • Urine and serum toxicology panels
    • A thorough medication and social history may provide enough information to determine the underlying cause of the priapism without collection of these studies.
      • Testing for potential substances may have a high rate of false negativity, particularly with synthetic and otherwise altered versions of common illicit substances
  • While CBC, hemoglobin electrophoresis, and toxicology panels may contribute to the identification of underlying cause, they often will not be used to guide treatment of the acute presentation

Imaging[edit | edit source]

Color Doppler US[edit | edit source]

  • Indications
    • Should not be incorporated into the acute evaluation and management of priapism in the emergency department by non-urologist specialists
      • Not the primary way to diagnose priapism
    • May be utilized in less clearly delineated cases to differentiate between acute ischemic priapism and non-ischemic priapism
  • Findings§
    • Acute ischemic priapism
      • Bilateral absence of flow through the cavernosal arteries
      • Peak systolic flows <50 cm/sec
      • Mean velocity <6.5 cm/sec
      • Diastolic reversal (i.e., negative end diastolic velocities)
    • Non-ischemic priapism
      • Peak systolic velocities of >50 cm/sec.
      • In the non-acute setting, may identify anatomical abnormalities, such as a cavernous artery fistula or pseudoaneurysm in patients who already have been diagnosed with non-ischemic priapism.
        • These abnormalities may occur following a straddle injury or direct scrotal trauma and are, therefore, most often found in the perineal portions of the corpora cavernosa.
  • Use is limited by
    • Number of specialists who can currently perform the procedure
    • Equipment might not be readily available

Pelvic MRI[edit | edit source]

  • Likely does not have a role in the initial diagnostic and treatment phase of priapism
  • Can be used to demonstrate:
    1. A well-established arteriolar-sinusoidal fistula
    2. Presence and extent of tissue thrombus
    3. Corporal smooth muscle infarction
    4. Corporal malignancy or metastasis

Penile arteriography[edit | edit source]

  • Too invasive as a diagnostic procedure to differentiate ischemic from non-ischemic priapism

Management[edit | edit source]

Ischemic Priapism[edit | edit source]

See AUA/SMSNA Guideline Flowchart on Management of Acute Ischemic Priapism

Patient Counselling[edit | edit source]

  • Natural history of untreated acute ischemic priapism is (2):
    1. Possible permanent loss of erectile function
      • ED is the most significant complication in patients with prolonged acute ischemic priapism.
    2. Penile shortening due to corporal fibrosis
  • All patients with persistent acute ischemic priapism should be counseled that there is the chance of erectile dysfunction
    • If acute ischemic priapism event >36 hours, likelihood of erectile function recovery is low.
    • In a patient with acute ischemic priapism >36 hours, surgical interventions, such as distal shunting, with or without tunneling, may be required to achieve detumescence; as it is unlikely the acute ischemic event will resolve with ICI therapy of phenylephrine and aspiration.
      • As the duration of the priapism increases, patients may be refractory to first-line treatments, such as ICI of phenylephrine and aspiration, with or without irrigation.

Non-surgical management[edit | edit source]

  • Conservative therapies (i.e., observation, oral medications, cold compresses, exercise) are not recommended in the management of acute ischemic priapism
    • Minimal corporal blood flow in priapism limits of oral agents
    • Cold compresses should never be used in persons with SCD to avoid provoking vasoconstriction and intravascular sickling
  • First-line: intracavernosal phenylephrine AND corporal aspiration, with or without irrigation
    • Clinicians treating acute ischemic priapism may elect to proceed with alpha adrenergics, or aspiration and saline irrigation, or a combination of both therapies based on their clinical judgment
      • ICI with phenylephrine should begin as rapidly as possible following diagnosis
        • These statements are taken near verbatim from AUA guidelines, but are conflicting
        • Intracavernosal treatments should not be delayed due to other systemic therapies (e.g., hydration, exchange transfusion), but may be administered concomitantly in most cases.
        • Even in cases where preserved erectile function is unlikely, clinicians may elect to perform combined treatments to improve penile pain, if present. 
        • Intracavernosal therapies may be deferred when ED is anticipated, and expedited placement of a penile prosthesis is planned.
Intracavernosal α-adrenergic injection[edit | edit source]
  • MOA of α-agonists: vasoconstrictors (cause smooth muscle contraction of the cavernous artery and arterioles)
  • Options: phenylephrine, etilefrine, ephedrine, epinephrine, norepinephrine, metaraminol)
    • Phenylephrine is the α-agonist of choice in ischemic priapism
      • Has less systemic side effects compared to other alpha-agonists due to its relatively selectivity for α1-adrenergic receptors with minimal β-mediated ionotropic and chronotropic cardiac effects
      • Use in this context is off-label
Technique[edit | edit source]
  • The optimal regimen for phenylephrine dosing, frequency, and method of administration has not been clearly defined
    • Phenylephrine diluted in normal saline to a concentration of 100-500 mcg/mL (0.1-0.5 mg/mL) (optimally premixed by pharmacy to minimize risks of miscalculation/overdose)
      • Dilute 10mg/mL vial of phenylephrine with 19mL of normal saline to get 10mg/20mL volume = 0.5mg/mL = 500mcg/mL
    • Administered intracavernosally (not subcutaneously)
    • Administered laterally (3 or 9 o’clock position) near the base of the penile shaft
      • Small needles may be used (e.g., 27G) if not aspirating
    • Doses administered ≥5 minutes apart
    • May be continued for up to 1 hour
      • If the erection persists despite repeated attempts with injections and aspiration/irrigation > 1 hour, proceed with more definitive therapy (i.e., shunting procedure).
    • In cases where the combination of phenylephrine and aspiration/irrigation are performed, aspiration should precede phenylephrine administration to permit fresh, oxygenated blood to fill the corpora and potentially improve the yield of phenylephrine administration
      • The penis is aspirated between successive injections by tightly pinching the shaft at the penoscrotal junction, just below the site of needle insertion
    • No recommendations can be made about maximum safe dosage.
      • Although there is no upper limit to the number of injections which may be performed, injections should be stopped if blood pressure changes are detected.
        • Hypertensive stroke has been reported as a complication of cumulative administration of 2 mg [20mL if 100 μg/mL; 10ml if 200 μg/mL]
Adverse effects of intracavernous sympathomimetics[edit | edit source]
  1. Hypertension (most common)
  2. Reflex bradycardia (most common)
  3. Tachycardia
  4. Irregular cardiac rhythms
  5. Headache
  6. Dizziness
  • Patients receiving intracavernosal injections with phenylephrine should be monitored for blood pressure and heart rate
    • Blood pressure and heart rate monitoring seems especially prudent in patients with a history of cardiovascular disease, hypertension, prior stroke, and those using medications such as monoamine oxidase inhibitors (MAOIs).
Contraindications[edit | edit source]
  • Relative: Use in patients on Monoamine Oxidate Inhibitors (MAOIs) (e.g. isocarboxazid, phenelzine, selegiline, tranylcypromine)
    • No reports of toxicity when used for priapism in males using MAOI
    • Potentiation of phenylephrine effects by prior administration of MAOI is most significant with use of oral phenylephrine, which is dissimilar from intracavernosal administration.
    • Gradual dose escalation may be reasonable when treating priapism in men using these medications.
Corporal aspiration, with or without irrigation[edit | edit source]
  • Corporal aspiration
    • Refers to the intracavernosal placement of a needle followed by withdrawal of corporal blood.
    • Will immediately soften the erection and relieve pain
    • May relieve priapism in 36% of cases
  • Corporal irrigation
    • Refers to subsequent instillation of fluid (typically saline) into the corpora after corporal aspiration.
  • Corporal aspiration and irrigation are often combined to remove clotted, deoxygenated blood and restore arterial flow and smooth muscle and endothelial function. 
    • They may be performed alone or combined with instillations of phenylephrine.
      • No studies have compared aspiration and irrigation with saline to alpha adrenergic injections alone
      • Studies suggest greater resolution rates with combination therapy of aspiration, irrigation, and intracavernosal alpha adrenergics compared to alpha adrenergics alone
  • Technique
    • Steps for aspiration/irrigation with phenylephrine administration:
      1. Perform a penile block with local numbing medication (if not previously performed).
      2. Place a 16-18 gauge butterfly needle in the 3 or 9 o’clock position (to avoid the dorsal neurovascular bundle) on the penis near the base.
      3. Connect the butterfly needle to a 30-60 cc Luer Lock syringe.
      4. Alternate between aspiration of blood clots and instillation of saline (chilled if available and if the patient does not have sickle cell disease) until some degree of detumescence can be achieved.
      5. Instill phenylephrine.
      6. Allow 3-5 minutes of time to pass.
      7. Repeat steps 4-6 until detumescence is achieved or until the decision has been made to proceed with surgical shunting.
      8. If temporary detumescence is achieved with aspiration followed by a rapid refilling of blood despite multiple attempts of phenylephrine instillation, consideration may be given to placement of a firm penile wrap at the time of aspiration to maintain detumescence.

Surgical management (Shunting)[edit | edit source]

  • Principle of shunt procedure is to reestablish corporal inflow by relieving venous outflow obstruction; this requires creation of a fistula between the corpora cavernosa and the glans penis, corpora cavernosa and corpus spongiousum, or corpora cavernosa and dorsal or saphenous veins.
Indications (3)[edit | edit source]
  1. Persistent acute ischemic priapism after intracavernosal phenylephrine and aspiration, with or without irrigation
    • Surgical shunting should not be performed until BOTH alpha adrenergics and aspiration and saline irrigation have been attempted
  2. Injections of sympathomimetics has resulted in a significant cardiovascular side effect
  3. Malignant or poorly controlled hypertension
  • Consider for ischemic priapism events ≤72 hours
    • In priapism lasting > 72 hours, consideration should be given to foregoing a shunt
Approach[edit | edit source]
  • Classification: distal vs. proximal
  • Distal (6)
    • Percutaneous (3) (through distal glans towards corpus cavernosum) WET
      1. Winter: large-bore needle or angiocatheter (least effective)
      2. Ebbehoj: straight incision with No. 11 blade
      3. T shunt: No. 10 blade is rotated 90° after insertion
        • After Ebbehoj or T shunt, the glans is sutured closed with absorbable suture. Discharge home if the penis remains flaccid for 15 minutes. If erection returns or persists, a second T shunt is recommended on the opposite side of the meatus.
    • Open (3): ATC
      1. Al -Ghorab: excision of a 5-mm circular cone segment of the distal tunica albuginea (see Figure in article)
      2. Tunnelling: modification of the Al-Ghorab; after excising the circular core of distal tunica albuginea, a 7/8 Hegar dilator is inserted down each corporal body through the tunica window
      3. Combined distal shunt and corporal tunneling maneuver
        • Addition of tunneling may afford slightly higher rates of successful detumescence, but associated with greater degradation of post-procedure erectile function compared to distal shunting alone
  • Proximal (3)
    • Options (open)
      1. Proximal corpus cavernosum to spongiosum shunt (Quackles); require a trans-scrotal or transperineal approach
      2. Proximal corpus cavernosum to saphenous vein shunt (Grayhack) - a wedge of tunica albuginea is removed and the vein is anastomosed end to side of corpora cavernosa.
      3. Proximal corpus cavernosum to deep dorsal vein shunt (Barry)
    • Represents a historical procedure and has largely been replaced by distal shunts with tunneling procedures
      • Technically more difficult to perform that distal, likely no surgeons who have extensive experience
    • Should not be considered a mandatory procedure for men who have been confirmed to have failed distal shunting
  • A distal corporoglanular shunt, with or without tunneling, should be performed in patients with persistent acute ischemic priapism
    • The optimal type of distal corporoglanular shunt has not been defined
    • Percutaneous distal shunting is less invasive than open distal shunting and can be performed with local anesthetic in the emergency department.
    • Open shunting procedures, especially those that require passage of dilators into the corpora cavernosa, will require general anesthesia and an operating room
  • Inadequate evidence to quantify the benefit of performing a proximal shunt (of any kind) in a patient with persistent acute ischemic priapism after distal shunting.
Adverse Events of Shunting (6):[edit | edit source]
  1. Penile edema
  2. Hematoma
  3. Infection
  4. Urethral fistula
  5. Penile necrosis
  6. Pulmonary embolism
Methods to prevent shunt obstruction and subsequent failure (3):[edit | edit source]
  1. Avoid compressive penile dressings
  2. Consider anticoagulation
  3. Patient should periodically squeeze and release the distal penis to “milk” the shunt maintaining patency

Resolution of acute ischemic priapism[edit | edit source]

  • Characterized by the penis returning to a flaccid, nonpainful state, with restoration of penile blood flow.
    • Oftentimes, persistent penile edema, ecchymosis, and partial erections occur and mimic unresolved priapism.
      • This often relates to the duration of priapism and may also signify segmental regions of cavernosal ischemia/necrosis.
  • After shunting, follow-up with the patient regarding erectile function and any subsequent ED therapies

Special Scenarios[edit | edit source]

Priapism refractory to shunting[edit | edit source]
  • A vascular study (such as a PDUS) or cavernosal blood gas should be performed prior to performing additional interventions (repeat distal or proceeding to proximal shunting).
    • Goal is to assess penile hemodynamic characteristics and extent of necrosis/fibrosis to differentiate persistent acute ischemic priapism from reactive hyperemia or conversion to non-ischemic priapism and inform secondary treatment decisions
      • Penile corporal blood gas can help with decision making about proceeding to additional surgical procedures including placement of an immediate penile prosthesis.
      • Penile duplex ultrasound
        • Can be used to
          • Differentiate between acute ischemic and non-ischemic priapism
          • Determine shunt patency by showing restoration of cavernosal arterial inflow in a patient who has undergone a distal shunt
    • Further surgical decisions should not be based only on exam
    • Evaluating the status of a patient with refractory priapism is particularly important in the event that a patient is referred from another institution and/or the clinician is seeing a patient who had been previously treated elsewhere and a complete patient history may not be available.
Acute Ischemic Priapism >36 hours[edit | edit source]
  • Options
    1. Observation, pain control, and outpatient follow-up
      • May bypass more invasive procedures (e.g., surgical shunting).
    2. Distal shunting, with or without tunneling
    3. Early implantation of penile prosthesis
  • Unlikely the acute ischemic event will resolve with ICI therapy of phenylephrine and aspiration.
    • The response to phenylephrine decreases with increased duration of priapism
  • It is important to note that before considering conservative management or penile prosthesis placement in men with a priapism >36 hours, the timeline should be sufficiently confirmed.
    • Patient histories relating to an exact timeline may often be unreliable, particularly in cases of concomitant substance use, episodes of intermittent detumescence, recurrent priapism (e.g., SCD), or partial (not fully rigid) erections. In these settings, clinical judgment is required to identify the true timeline for onset of ischemia (i.e., onset of severe, persistent penile pain). 
    • If the timeline is in question, clinicians should preferentially attempt to decompress the priapism, particularly in younger men or those with high baseline erectile function.
Early implantation (within 2 weeks) of penile prosthesis[edit | edit source]
  • Indications
    • May be considered in a patient with untreated acute ischemic priapism > 36 hours or in those who are refractory to shunting, with or without tunneling.
  • Advantages to prostheses placed in the setting of acute ischemic priapism (5):
    1. Detumescence
    2. Relief of pain
    3. Preservation of penile length
    4. Return to sexual activity
    5. Overall satisfaction
  • Disadvantages to immediate implantation (3):
    1. Urologist involved for may lack the experience, comfort level, or materials to render device placement practical and/or possible
    2. Repetitive bedside irrigation procedures may, in theory, increase the chances for bacterial entry into the corpora that could threaten an implant with infection.
      1. Infection rates < 10%, higher with delayed implantation,★
    3. Distal shunts may have compromised the integrity of the tunica albuginea that would surround an implant, possibly predisposing to erosion.
Recurrent Ischemic Priapism[edit | edit source]
  • Options to prevent subsequent episodes
    • Idiopathic (7):
      1. Ketoconazole with prednisone
        • Highest success rate
        • Adverse effects
          1. Liver toxicity, thus warranting frequent assessment of liver function tests
      2. Pseudoephedrine, an oral α-adrenergic agonist, promotes muscle contraction within the erectile tissue
      3. Phosphodiesterase type 5 inhibitors
      4. Aspirin
      5. Baclofen
      6. Dutasteride
      7. Cyproterone acetate
        1. Anti-androgen
        2. Not available in the United States
    • Associated with sickle cell disease
      1. Same options as idiopathic
      2. Etilefrine
      3. Hydroxyurea
      4. Automated exchange transfusion
    • Drug therapy is typically initiated at bedtime
    • Patients with recurrent ischemic priapism should be informed that hormonal regulators (ketoconazole, cyproterone acetate) may impair fertility and sexual function
      • Adverse effects, due to manipulation of hypothalamic-pituitary-gonadal axis
        • Fatigue
        • Hot flashes
        • Breast tenderness
        • Changes in mood
        • ED
        • Negatively impact sperm parameters
    • Home self-injection of intracavernous α-adrenergic agent
      • Not a preventative strategy; may avert a full-blown episode of ischemic priapism when administered at home for prolonged morning erections
    • Optimal strategy is unknown
Sickle Cell Disease and other Hematologic Disorders[edit | edit source]
  • The best intervention is to relieve episodes with prompt intracavernosal phenylephrine and corporal aspiration, with or without irrigation, as in other acute ischemic priapism patients, before proceeding to systemic therapies specific to the underlying disorder
  • Standard sickle cell assessment and interventions should be considered concurrent with initiation of urologic intervention. Specifically, disease specific systemic care should address:
    • Hydration with IV fluid only if made NPO (maintenance rate) or dehydrated (replace deficit plus maintenance rate)
      • Hyperhydration is not indicated and may predispose to acute chest syndrome.
    • Supplemental oxygenation only if hypoxic.
    • Pain management with oral or parenteral opioids as per usual painful events (remembering that some patients with SCD may be tolerant to analgesia because of those prior experiences).
    • Hematologic status comparison of CBC and reticulocyte count to baseline values
      • Best done in consultation with the patient’s hematologist.
      • Transfusion is not indicated if hemoglobin is near usual value, and over-transfusion may be associated with neurologic events.
      • Acute exchange transfusion is not indicated.
      • If operative shunting procedures are required, consideration should be given to a simple transfusion of packed red blood cells to raise the hemoglobin to 9-10 g/dl prior to general anesthesia
      • Rarely are blood products required before an aspiration and irrigation procedure, the one exception may be with a very low platelet count (<20,000/uL).
    • Presence of other acute sickle cell events: neurologic disorders including acute stroke, acute chest syndrome, biliary colic, renal insufficiency which while not associated with a higher frequency of priapism may present at the same time.
  • Ice packs and other cold compresses should never be used in SCD patients as they may worsen painful events by precipitating intravascular sickling.
  • Most patients with SCD experience recurrent short ischemic priapism events, (lasting <4 hours and commonly referred to as stuttering priapism) but acute episodes and particularly recurrent acute episodes occur commonly enough (both before and after shorter, stuttering events) that education about when to seek urologic attention is a critical part of the patient education in SCD disorders.
  • Drugs that have been tried to prevent subsequent priapism episodes
    • Etilefrine
    • Ephedrine
    • Pseudoephedrine
    • Terbutaline
    • PDE5is (e.g., sildenafil, tadalafil)
    • 5 alpha reductase inhibitors (dutasteride or finaseride)
    • Anti-androgens (cyproterone, bicalutamide, leuprolide, stilboesterol)
    • Ketoconazole/prednisone
    • Hydroxyurea
  • Ongoing chronic (monthly) exchange transfusions do appear to be associated with a reduction in acute and stuttering priapism episodes.
    • Exchange transfusion should not be as the primary treatment in patients with acute ischemic priapism associated with sickle cell disease.
    • For prolonged acute priapism events that cannot be relieved with intracavernosal phenylephrine and corporal aspiration, exchange transfusion can be considered
Prolonged Erection Following Intracavernosal Vasoactive Medication[edit | edit source]
  • A prolonged erection (≤4 hours in duration) following iatrogenic- or patient self-administration of erectogenic medications into the corpus cavernosum (ICI) represents a distinct pathology when compared to acute ischemic priapism or non-ischemic priapism.
    • Much more common and may be managed differently than acute ischemic priapism
  • Management
    • See AUA/SMSNA Guideline Flowchart on Management of Prolonged Erection
    • If erection > 4 hours, treat according to ischemic priapism algorithm
      • All patients should be instructed at the time of ICI training, or after receiving an in-office erectogenic therapy, that they should return to either the office or emergency department if they experience an erection lasting > 4 hours.
    • If prolonged erection 1-4 hours after home ICI or following an in-office injection
      • Options
        1. Conservative options (in the case of home ICI) including ice compresses to the penis, laying supine, ejaculation, exercise, penile compresses and oral medications such as pseudoephedrine, in the absence of any clinical data demonstrating efficacy
          • Utilize intracavernosal phenylephrine if conservative management is ineffective in the treatment of a prolonged erection.
        2. In-office phenylephrine
          • Not appropriate for clinicians who administer in-office erectogenic medications to refer the patient to the emergency department as a matter of routine following an in-office injection, rather, the patient should return to the office for detumescence whenever possible.
          • Intracavernosal aspiration and irrigation likely represents too aggressive of a therapy for this specific clinical scenario to be used as a first-line therapy
            • Intracavernosal phenylephrine can be delivered through a small needle (27G) compared to needle typically used for aspiration/irrigation (16 or 18G)
            • The physiologic rationale for aspiration and irrigation is to remove intracavernosal clots and permit entry of fresh blood in an attempt to restore smooth muscle function and vascular drainage.  As the pathologic state of intracavernosal clotting and ischemia likely is not present with prolonged erections <4 hours, aspiration and irrigation is rarely warranted.
              • Persistent, prolonged erections may be considered for aspiration and irrigation if phenylephrine alone is unsuccessful.
      • Duration of a persistent erection requiring intervention is not clearly defined. Generally,
        • Erection lasting
          • <1 hour post injection would not require intervention
          • >4 hours would warrant treatment, regardless of underlying etiology
          • Decision to intervene in the time-period between 1 and 4 hours would depend on several clinical factors:
            1. Penile rigidity
              • Mild erection (i.e., not sufficient to penetrate without assistance) would not require treatment, whereas a fully rigid erection might
              • Intermittently rigid erection is considered differently than a fully rigid erection, which has remained persistent since the original injection
            2. Specific medication used and dosage
              • Alprostadil alone is likely associated with shorter durations of erections and likely has a lower risk of ischemic priapism compared to combination therapies, which include  papaverine and/or phentolamine
              • Higher dosages are empirically more likely to result in a prolonged erection compared to lower ones
            3. Age
            4. Baseline erectile function
            5. Reliability/capacity
            6. Comorbid conditions
            7. Pain as an indicator for treatment is not relevant in many scenarios, as the intracavernosal medications themselves are often associated with penile pain

Non-ischemic priapism[edit | edit source]

  • See AUA/SMSNA Guideline Flowchart on Management of Non-ischemic Priapism
  • Cavernous aspiration has only a diagnostic role in nonischemic priapism.
    • Repeated aspirations, injection, and irrigation with intracavernous sympathomimetics have no role in the treatment of nonischemic priapism.
  • First-line: observation
    • Non-ischemic priapism is not an emergency; initial observation is recommended
    • Spontaneous resolution or response to conservative therapy has been reported in up to 62% of cases
      • No comparative studies of intervention vs. conservative management in non-ischemic priapism
    • Conservative measures include ice applied to the perineum and site-specific compression
    • 4-week period is reasonable, unless the patient is severely bothered by the tumesced penis
      • After the 4-week mark, the patient’s fistula can be re-evaluated using penile duplex ultrasound; the patient’s sexual function and degree of bother can be further quantified. In cases where the fistula is unchanged and/or where patient bother is significant, intervention may be considered.
    • Consider penile duplex ultrasound for assessment of fistula location and size in a patient with diagnosed non-ischemic priapism
      • Screening for anatomical abnormalities and identification of cavernous artery fistula (turbulent flow may be detected) or pseudoaneurysm location and size
      • Ultrasonography is of particular benefit in a patient with NIP being considered for fistula embolization. T
        • Allows for communication between the urologist and radiologist prior to intervention regarding fistula location, size, and eventual choice of vascular access.
      • Ultrasonography may also potentially help with the follow-up of a patient with NIP opting for observation through tracking of fistula and its size.
  • Second-line: percutaneous fistula embolization
    • Indications
      • Persistent non-ischemic priapism who have failed a period of observation and are bothered by persistent penile tumescence, and who wish to be treated
    • Prior to embolization
      • Fistula should be readily visible on a PDUS.
      • Patients should be informed that embolization carries a risk of erectile dysfunction, recurrence, and failure to correct non-ischemic priapism.
        • Pooled analysis suggest that embolization resulted in penile detumescence in 85% of patients, with 80% of men retaining functional erections
          • Bilateral arterial embolization significantly increased the risk of ED.
    • Embolization should only be attempted by an experienced interventional radiologist.
    • In patients who have failed an initial attempt at embolization, patients should be offered a second attempt at an embolization procedure
      • Embolization of non-ischemic priapism may require retreatment; a single treatment of embolization carries a recurrence rate of 30%.
      • Second attempt at an embolization procedure likely to be more effective and safer than an attempt at surgical ligation, given the lack of experience in the latter approach for most urologists and the poor data supporting ligation.
  • Surgery
    • Surgical ligation of the corporo-cavernosal fistula following failed attempts at embolization (or when embolization is not available at the center treating the patient) is an option for patients with non-ischemic priapism
    • The lack of familiarity of most urologists with this surgical approach makes the procedure particularly challenging
      • Surgical approach is transcorporal

Questions[edit | edit source]

  1. What is the definition of priapism? What are the subtypes?
  2. What proportion of patients will recover erectile function for priapism reversed within a) 12 hours b) 12-24 hours c) 24-36 hours d) >36 hours
  3. After what duration of priapism is shunting no longer recommended?
  4. List risk factors for priapism.
  5. Which medications are associated with risk of priapism?
  6. What are the initial investigations in a patient with suspected priapism?
  7. What is the expected PO2, PCO2, and pH of normal arterial gas vs. mixed venous gas vs. corporal aspirate from ischemic priapism?
  8. What is the maximum dose of phenylephrine that should be administered?
  9. What are potential adverse effects related to phenylephrine injection?
  10. List and briefly describe the different shunting procedures
  11. What are potential complications of shunting?
  12. What are potential advantages to immediate implantation of prosthesis in the management of priapism?
  13. What medication can be used for prophylaxis in stuttering priapism? What are other options?

Answers[edit | edit source]

  1. What is the definition of priapism? What are the subtypes?
    • Definition: a full or partial erection that continues > 4 hours beyond sexual stimulation and orgasm or is unrelated to sexual stimulation
    • Subtypes: ischemic, stuttering, non-ischemic
  2. What proportion of patients will recover erectile function for priapism reversed within a) 12 hours b) 12-24 hours c) 24-36 hours d) >36 hours
    1. 100%
    2. ≈80%
    3. ≈45%
    4. 0%
  3. After what duration of priapism is shunting no longer recommended?
    • >72 hours
  4. List risk factors for priapism.
  5. Which medications are associated with risk of priapism?
  6. What are the initial investigations in a patient with suspected priapism?
    • H+P, CBC, coagulation profile, blood gas, +/- urine toxicology screen, +/- sickle cell preparation and electrophoresis
  7. What is the expected PO2, PCO2, and pH of normal arterial gas vs. mixed venous gas vs. corporal aspirate from ischemic priapism?
    • Normal arterial blood gas: PO2 > 90, PCO2 <40, pH 7.4
    • Mixed venous gas: PO2 40, PCO2 50, pH 7.35
    • Ischemic priapism: PO2 < 30, PCO2 >60, pH < 7.25
  8. What is the maximum dose of phenylephrine that should be administered?
    • 2mg
  9. What are potential adverse effects related to phenylephrine injection?
    1. Headache
    2. Dizziness
    3. Hypertension
    4. Reflex bradycardia
    5. Tachycardia
    6. Irregular cardiac rhythms
  10. List and briefly describe the different shunting procedures
    • Distal
      • Percutaneous
        1. Winter
        2. Ebbehoj
        3. T-shunt
      • Open
        1. Al-Ghorab
        2. Corporal snake
        3. Combined T-shunt and Corporal Snake
    • Proximal
      1. Cavernosum-spongiosum
      2. Cavernosum-saphenous/deep dorsal vein
  11. What are potential complications of shunting?
    • Penile edema, hematoma, infection, urethral fistula, penile necrosis, and pulmonary embolism
  12. What are potential advantages to immediate implantation of prosthesis in the management of priapism?
    1. Corporal fibrosis not yet established
    2. Penile length may be preserved
  13. What medications can be used for prophylaxis in stuttering priapism?
    1. Phenylephrine
    2. Oral beta agonist
    3. PDE5 inhibitor
    4. GnRH or antiandrogens
    5. Intracavernosal alpha-agonist

References[edit | edit source]